Den 32-gradiga (°C) atmosfärseffekten

Gästinlägg 24 av Gösta Pettersson andra delen av två.  För del 1, Se gårdagens inlägg – ”Den gravito-termala effekten”


FN:s klimatpanel IPCC är ett politiskt organ som i huvudsak består av diplomatiska representanter för regeringarna i FN:s medlemsstater. IPCC gav i Introduktionen till sin rsta rapport år 1990 politisk sanktion åt begreppet växthuseffekt (GreenHouse Effect, GHE). Där redogjorde man för och anammade det synsätt som utgör grunden för GHE-proponenternas hypoteser:

Troposfären innehåller s. k. växthusgaser, dvs. gaser som förmår absorbera infraröd värmestrålning. Jordytans värmeutstrålning absorberas till stor del av sådana växthusgaser. Dessa återutsänder sedan den absorberade strålningsenergin i alla riktningar, vilket innebär att nästan hälften av den återstrålas till jordytan. Sådan absorption och återstrålning medför enligt IPCC att jordytan och den lägre atmosfären blir mycket varmare än vad den skulle ha varit om jordens atmosfär hade saknat växthusgaser.
Untitled

Bild 1.  IPCC:s schematiska illustration av växthuseffekten i 1990:års rapport

IPCC definierar alltså växthuseffekten som den förhöjning av jordytan och atmosfärens temperatur som har sin orsak i växthusgasernas specifika förmåga att absorbera och återutstråla värmestrålning. Effekten kallas naturligr den hänför sig till atmosfärens förindustriella sammansättning och förstärktr den förmodas härröra från den industriella tidens förhöjda halter av koldioxid och andra växthusgaser. Därefter ställer IPCC i första rapportens Introduktion (sid. xxxvi) frågan ”Hur vet vi att växthuseffekten är verklig?” och ger tre svar. Det första och mest (för att inte säga enda) beaktansvärda av dem lyder på engelska som följer:

Firstly, the mean temperature of the Earth’s surface is already 32 °C warmer than it would be if the natural greenhouse gases (mainly carbon dioxide and water vapour) were not present.

Hur har politikerna och deras vetenskapliga rådgivare kommit fram till denna axiomatiskt presenterade slutsats?Definitivt inte genom empiriska observationer, eftersom det inte existerar någon Jord med en atmosfär utan växthusgaser. Föreställningen om Jordens 32-gradiga naturliga växthuseffekt emanerar i stället från vad t. ex. SMHI kallar och redovisar som ett tankeexperiment (länken laddar ner en pdf-fil), nämligen en kalkyl av vilken global yttemperatur vår planet skulle förväntas uppvisa om den hade saknat atmosfär. Kalkylen grundar sig på det rimliga antagandet att Jordens yttemperatur huvudsakligen bestäms av balansen mellan den strålningsenergi Jorden tar upp från solen respektive avger till rymden.

Solen skiner på Jorden med en intensitet given av den så kallade solarkonstanten (S). Denna är inte  helt konstant, men har genomsnittligt värdet 1370 watt per kvadratmeter vid atmosfärens utkant. Detta värde kan som en god approximation även tas som instrålningsvärdet för en tänkt atmosfärsfri Jord med radien R och därmed den strålningsmottagande tvärsnittsytan πR2. På grund av jordens vithet (albedo = 0,30) reflekteras 30% av den infallande strålningen, varför endast 70% tas upp av jordytan. Mängden upptagen strålningsenergi ges alltså av

         Absorberat  =  0,7 S (πR2)                                                     (1)

Strålningsenergin som utsänds från den tänkta atmosfärsfria och isotermiska (samma temperatur överallt) Jorden är enligt Stefan-Boltzmanns strålningslag proportionell mot fjärdepotensen av dess temperatur T och avges från hela dess yta 4πR2. Mängden avgiven strålningsenergi ges då av      

         Emitterat  =    σ T4 (4πR2)                                                      (2)

där σ står för Stefan-Boltzmann konstanten och har värdet 0,0000000567 watt per kvadratmeter.

Vid strålningsbalans ska Absorberat vara lika med Emitterat. Yttemperaturen av tankeexperimentets atmosfärsfria Jord bestäms därför av ekvationen

         0,7 S  =  4σT4                                                                        (3)

vilken har lösningen T = 255 K (-18 °C).

IPCC ansåg i sin första rapport Jordens globala yttemperatur vara 14 °C, vilket enligt tankeexperimentet innebär att atmosfären förhöjer jordytans temperatur med 32 °C. Det är denna tankemässigt kalkylerade atmosfärseffekt som IPCC betraktar som en ”naturlig växthuseffekt”.

Det är ett betraktelsesätt som en mångfald GHE-skeptiker rest starka invändningar mot och som även jag själv finner märkligt, alldenstund atmosfären endast till en ringa del (normalt mindre än 2%) består av växthusgaser. IPCC ger ingen motivering för sin bedömning, men i debatten om tankeexperimentet har GHE-proponenter framfört följande argument till stöd för IPCC:s synsätt:

En atmosfär utan växthusgaser kan inte stråla ut värme mot rymden, utan värmeutstrålningen måste ske från jordytan. I en tänkt frånvaro av atmosfäriska växthusgaser måste alltså Jorden strålningsmässigt bära sig åt som om den helt hade saknat atmosfär och antaga yttemperaturen -18 °C. Den 32 °C högre yttemperatur som Jorden med sin växthusgasinnehållande atmosfär i realiteten befunnits uppvisa måste därför representera en växthuseffekt. 

Det är inte någon hållbar slutsats. Må vara att närvaron av växthusgaser måhända krävs för att atmosfären ska kunna ge någon förhöjning av jordens yttemperatur överhuvud taget. Men därav följer inte att den kalkylerade 32-gradiga atmosfärseffekten till hundra procent måste anses representera en växthuseffekt. Ytterligare argument krävs för att avgöra hur mycket av den beräknade 32-gradiga temperaturförhöjningen som kan förmodas hänföra sig till en strålningsrelaterad växthuseffekt och hur mycket som kan förmodas hänföra sig till de temperaturpåverkande sekundäreffekter som måste uppkomna i en atmosfär som innehåller växthusgaser.

Närvaron av växthusgaser i atmosfären har nämligen en drastisk effekt på värmetransporten från Jorden till rymden. Värmeutstrålningen behöver inte längre ske från jordytan, utan kan även ske från atmosfären. Jordytans temperatur kommer därför att vara beroende av ett flertal avkylande processer utöver värmeutstrålning, t. ex. av värmetransport till atmosfären genom ledning, konvektion och avdunstning av vatten från hydrosfären. Och med värmeutstrålning från atmosfären blir även den gravito-termala effekten av intresse som en väsentlig temperaturpåverkande faktor.

Det diskuterade tankeexperimentet kan förvisso tas som belägg för att det föreligger en 32-gradig atmosfärseffekt. Men vill man i likhet med IPCC (samt SMHI och GHE-proponenter i allmänhet) hävda att detta är en växthuseffekt, så krävs det vetenskapligt betydligt mera övertygande argument än det trosvissa påståendet att så är fallet.

Låt oss nu jämföra tankeexperimentets värld med verkligheten. Solen värmer jordytan. Stjärnorna är också solar i generell bemärkelse, men befinner sig på så stort avstånd att deras bidrag till Jordens uppvärmning kan betraktas som försumbart i det sammanhang som mitt inlägg berör. Jag är också i det sammanhanget beredd att acceptera IPCC:s bedömning att värmealstring genom radioaktivt sönderfall av material i Jordens inre inte signifikant bidrar till att värma upp jordytan. Jag utgår med andra ord i princip från att jordytan uppvärms av solen och inget annat.

Solens uppvärmning av Jorden balanseras av en utstrålning av värme till rymden, vilken enligt SMHI (samma länk som ovan) ”sker från högre nivåer upp i atmosfären, i stället för från jordens yta som får motta det mesta av den inkommande solstrålningen. I snitt ligger dessa nivåer i jordatmosfären på 4-8 km höjd där temperaturen är runt minus 19 grader, vilket motsvarar jordens effektiva utstrålningstemperatur i det tidigare diskuterade tankeexperimentet.

Minus 19 grader säger SMHI. Minus 18 grader kom IPCC fram till i tankeexperimentet, och den senare skattningen kommer jag att hålla mig till. Skillnaden på en grad återspeglar gissningsvis att SMHI använt ett något högre värde än IPCC för Jordens albedo och är tämligen ointressant. Den väsentliga informationen SMHI förmedlar är att Jordens värmeutstrålning mot rymden huvudsakligen sker från de luftskikt i mellersta troposfären där temperaturen genomsnittligt stämmer väl överens med den som enligt tankeexperimentet krävs för att skapa balans mellan in- och utstrålning för en himlakropp på Jordens avstånd från solen. Dessa utstrålningsskikt kommer jag att tankeexperimentellt medelvärdesutjämna och globalt generalisera till ett utstrålningsskikt, med acceptans av GHE-proponenternas förmodande att skiktet uppvisar den balansskapande temperaturen -18 °C.

Alla lär vara överens om att full förståelse av energiflödet från solen till Jorden och därifrån vidare ut i rymden kräver beaktande av värmetransporten från jordytan till utstrålningsskiktet. Hur stor del av den senare värmetransporten sker genom strålning, ledning, konvektion respektive processer som vattenavdunstning och därav föranledd transport av så kallad latent värme? Det är vetenskapligt intressanta frågor som meteorologer sedan länge beaktat och i ett flertal avseenden gett trovärdiga svar på.

Men beträffande detta inläggs rubricerade tema är detaljmekanismerna för värmetransporten från jordytan till utstrålningsskiktet av underordnad betydelse. Vad man överordnat kan konstatera är att det finns en värmetransportsom leder till en balanserad situation med ett utstrålningsskikt vars medeltemperatur är sådan att den normalt medför att Jordens energiavgivning blir lika stor som dess energiupptag från solen.

Då måste det (åtminstone tankeexperimentellt) även finnas en motsvarande balansskapande medeltemperatur för jordytan och den ytnära atmosfären. Vilken den temperaturen är avgörs av den troposfäriska temperaturgradientens lapse rate. Det är nämligen gradientens lapse rate som bestämmer vilken temperaturskillnad det kommer att föreligga mellan jordytan och utstrålningsskiktet på den höjd det senare hamnar vid balans mellan instrålning och utstrålning av energi till Jorden. När IPCC och GHE-proponenter hävdar att den 32-gradiga atmosfärseffekten utgör en växthuseffekt, så är det ekvivalent med att hävda att den troposfäriska temperaturgradienten helt och hållet återspeglar en växthuseffekt.

Att så skulle vara fallet saknas det någon som helst vetenskapligt befogad anledning att tro, vilket gårdagens KU-inlägg om den gravito-termala effekten syftade till att klargöra. 1870-talets termodynamiker (med Maxwell och Loschmidt i spetsen) har gett en övertygande teoretisk förklaring till varför troposfären uppvisar en temperaturgradient med de karakteristika den inom ramen för experimentell precision empiriskt befunnits uppvisa. Och efterföljande forskning har bekräftat att gradienten, oberoende av dess luftfuktighetsberoende lapse rate, representerar en gravitationell effekt. De teoretiska samband som härletts och allmänt accepterats inom meteorologin beskriver såväl ”torradiabatens” som ”våtadiabatens” lapse rate som proportionell mot gravitationskonstanten.

Maxwell-Loschmidts teoretiska analyser av den gravito-termala effekten förklarar temperaturgradientens existens och lapse rate. Däremot säger den gravito-termala teorin inget om vilken temperatur på jordytan gradienten ska räknas från. Den säger därmed ej heller något om på vilken höjd över jordytan utstrålningskiktet med sin balansskapande temperatur -18 °C kommer att återfinnas. Men teorin leder till den ofrånkomliga slutsatsen att var än utstrålningsskiktet befinner sig, så kommer dess temperaturskillnad gentemot jordytan att bestämmas av den troposfäriska temperaturgradientens lapse rate.

Med det sagt kan jag mera precist beskriva GHE-opponenters argumentering beträffande den 32-gradiga atmosfärseffekten. Antag, i enlighet med meteorologers beskrivning av kunskapsläget, att lufttemperaturen i troposfären avtar linjärt med ökande höjd från jordytan upp till tropopausen med en lapse rate som är gravitationellt bestämd, dvs. att standardatmosfärens ”våtadiabat” uppkommer på grund av den gravito-termala effekten och svarar mot det som Maxwell kallade en ”konvektionell termisk jämvikt”. Då måste temperaturskillnaden mellan vilka som helst två höjdmässigt skilda punkter på grafen för våtadiabaten återspegla en gravitationell effekt och inget annat. Varav som specialfall följer att den 32-gradiga temperaturskillnaden mellan utstrålningsskiktets balansskapande -18 °C och jordytans 14 °C återspeglar en gravitationell effekt utan någon som helst anknytning till vissa luftmolekylers speciella förmåga att absorbera infraröd värmestrålning.

Följaktligen finns det enligt detta argument ingen som helst anledning att fästa tilltro till IPCC:s och andra GHE-proponenters förmodande att den tankeexperimentellt påvisade 32-gradiga atmosfärseffekten representerar en växthuseffekt.

Slutsatsen att det saknas hållbara belägg för existensen av en 32-gradig ”naturlig växthuseffekt” har på sådana grunder dragits av ett stort antal klimatologiska forskare. Ett tjugotal av dem namnges i en artikel på den skeptiska bloggen The Hockey Schtick. Artikelförfattaren presenterar argument av statistisk-mekanistiskt ursprung, vilka leder till slutsatsen att utstrålningsskiktet med den balansskapande temperaturen -18 °C bör återfinnas på 5 km:s höjd över jordytan. Med följdslutsatsen att jordytan och den ytnära luften för standardtroposfären med dess empiriskt skattade lapse rate 6,5 °C/km bör uppvisa  5 x 6,5 = 32,5 °C högre temperatur. Artikelförfattaren hävdar därför att den 32-gradiga atmosfärseffekten kan teoretiskt härledas och förklaras utan något som helst hänsynstagande till vad GHE-proponenter kallar växthusgasers ”radiative forcing”, dvs. växthuseffekten.

Empiriskt kan man resonera omvänt. Vi vet (enligt IPCC:s skattning) att jordytans och den ytnära luftens temperatur är 14 °C. Det innebär för standardatmosfärens del (lapse rate 6,5 °C/km) att utstrålningsskiktet med dess balansskapande temperatur -18 °C bör finnas på (14+18)/6,5 ≈ 5 km:s höjd, konsistent med SMHI:s icke-medelvärdesutjämnade skattning 4–8 km:s höjd. Men framför allt belyser även det empiriska resonemanget att den observerade 32-gradiga temperaturskillnaden mellan utstrålningsskiktet och jordytan bestäms av den troposfäriska temperaturgradientens lapse rate. Godtar man GHE-opponenternas ovan angivna antagande att meteorologer korrekt beskrivit det föreliggande kunskapsläget, så ger även empiriska data klara belägg för att det existerar en observationsmässigt befäst 32-gradig atmosfärseffekt som är av gravitationellt ursprung.

Inom nutida klimatologisk forskning godtar man emellertid inte antagandet att troposfärens temperatur avtar linjärt från jordytan ända upp till tropopausen. 2021 år Nobelpris i fysik tilldelades bland andra Syukuro Manabe med hänvisning till hans modellberäkningar av växthuseffekten. Pehr Björnbom har i ett

KU-inlägg daterat 2012-06-01 uppmärksammat den ”konvektiva-radiativa” modell som Manabe och medarbetaren Wetherald 1967 lade fram som en mekanistisk förklaring till uppkomsten av växthuseffekter. För lufthalter av koldioxid lika med eller högre än 300 ppm framställer modellen det troposfäriska temperaturavtagandet som linjärt och styrt av Maxwells ”konvektiva termiska jämvikt” (dvs. av standardatmosfärens våtdiabat) upp till 8 km;s höjd över jordytan. På höjder överstigande 8 km föreskriver modellen att en radiativ termisk jämvikt tar över, med påföljd att temperaturavtagandets linearitet upphävs på sådant sätt att jordytans temperatur kan förhöjas genom en radiativ växthuseffekt.

Atmosfärsskiktet som uppvisar den gentemot instrålningen av energi till jorden balansskapande utstrålningstemperaturen -18 °C återfinns enligt Manabe-Wetheralds modell på lägre höjd än 8 km, dvs. i en region där den ”konvektiva jämvikten” förutsätts råda. Skiktets temperaturskillnad relativt jordytan återspeglar därför enligt modellen den gravito-termala effekten så som den t. ex. uttrycks av ”våtadiabaten” för standardatmosfären. Även den Nobelprisade Manabes ”konvektiva-radiativa” modell för växthuseffekten framställer alltså den 32-gradiga temperaturskillnaden mellan jordytan och troposfärens uttrålningsskikt som en gravitationell effekt.

Så sammanfattningsvis är den enda hållbara slutsatsen rörande detta inläggs rubricerade tema att tankeexperimentets 32-gradiga atmosfärseffekt måste anses utgöra en gravito-termal effekt. Såväl teoretiska analyser som empiriska observationer ger belägg för att det existerar  en gravitationsbetingad 32-gradig atmosfärseffekt som återspeglar luftmolekylernas massa och värmekapacitet, utan någon som helst påvisad kvantitativt signifikant relation till molekylernas spektrala egenskaper och förmåga att absorbera värmestrålning.

Det är en väsentlig slutsats, eftersom man från den kan gå ett steg längre och konstatera att den 32-gradiga ”naturliga växthuseffekt” som IPCC anser föreligga i själva verket tankeexperimentellt har befunnits vara av insignifikant storlek. Det följer av att den 32-gradiga atmosfärseffekten enligt ovan anförda argument helt och hållet kan hänföras till den gravito-termala effekten.

Kommentarer

Kommentera längst ner på sidan.

  1. Magma

    Jag är inte tillräckligt insatt i den del av atmosfärens fysik som påverkar jordens temperatur för att varken gå in och ifrågasätta eller för att applådera … men jag är en stark anhängare av synsättet att varje (ok, ordet ”varje” kan dock med fog ifrågasättas) ifrågasättande av vetenskapliga teorier bidrar till att öppna diskussioner som bidrar till att öka kunskapsnivån – ibland även i forskningens framkant.
    Jag ser fram emot att ta till mig bloggens synpunkter i kväll efter jobbet …

  2. Lennart Bengtsson

    1 Som lätt inses från vertikalkomponenten av Navier-Stokes rörelseekvation gäller att vid en atmosfär i vila befinner sig den vertikala tryckkomponenten i full balans med gravitationskraften. Vidare kan atmosfären betraktas som en ideal gas och då gäller den allmänna gaslagen. Från dessa två naturlagar följer att temperaturen avtar med höjden. För torr luft uppgår temperaturavtagande med ca 9°C per kilometer. Samma gäller för andra relevanta planeter som t ex Venus.
    2. Det är alldeles klart att om av någon anledning atmosfären inte skulle uppträda som en ideal gas och med konstant temperatur i vertikal led då skulle givetvis ”växthuseffekten” försvinna. Men varför göra en fråga av något som ändå är fullkomligt orimligt.
    3. Det är knappast meningsfullt att diskutera grundläggande fysik om läsarna är fullständigt okunniga om naturlagarna och dess matematiska form. Tyvärr förefaller även naturlagarna vara okunniga även för landets journalister och klimatdebattörer. Varför inte föreslå att de sätter sig på sin bak och lär sig lite elementa i fysik i stället för att prata fullständigt strunt. En annan möjlighet att syssla med något som man begriper som att sätta potatis eller steka köttbullar.

  3. JonasW

    Tycker man blandar ihop två olika saker:
    – att gravitationen medför att det finns en temperaturgradient i atmosfären
    – att utstrålningen sker från 5-10 km höjd

    Det är ingen naturlag att utstrålningen ska ske från hög höjd. Om atmosfären inte innehöll drivhusgaser och moln, så skulle utstrålningen från markytan obehindrat passera genom atmosfären, d.v.s. utstrålningen sker från höjden = 0 meter.
    Även i detta fall skulle man ha en temperaturgradient i atmosfären. Det finns inget samband mellan temperaturgradienten och utstrålningshöjden, d.v.s. de beror inte av varandra.

    Däremot innebär drivhusgaserna att atmosfären är relativt ogenomskinlig inom vissa frekvensband. Utstrålning kan bara ske när dessa gaser tunnats ut så mycket att strålningen kan nå rymden. Konkret får man alltså utstrålning från höga höjder (där det är kallare), vilket leder till att nettoutstrålningen minskar i dessa frekvensområden.

    En annan kommentar är att beräkningen av jordens jämviktstemperatur utan atmosfär (-18C) bygger på att jorden är jämnvarm. D.v.s. man får strålningsbalans vid -18C.

    Om man istället räknar med lokal strålningsjämvikt, vilket är det fysikaliskt korrekta, så blir polerna kallare och ekvatorn varmare. P.g.a. olineariteten i Stefan-Boltzmanns lag (temperaturen upphöjt till 4) kommer strålningsbalans inträffa vid en betydligt lägre medeltemperatur än -18C.
    Jämför med månen. Månen får lika mycket solljus som jorden (bortser från jordskugga). Månen är inte -18 C i medel. Månen är mycket kallare.

  4. Bjarne Bisballe

    ”Dessa återutsänder sedan den absorberade strålningsenergin i alla riktningar, vilket innebär att nästan hälften av den återstrålas till jordytan” .

    ”utan någon som helst påvisad kvantitativt signifikant relation till molekylernas spektrala egenskaper och förmåga att absorbera värmestrålning”.

    Men der er ikke nogen ir-varmestråling at absorbere, idet al ir-varmestråling er absorberet allerede i de nederste få hundrede meter over jordoverfladen og ved molekylekollisioner overgivet til de ’almindelige’ molekyler . Først flere kilometer oppe i atmosfæren er fotonudsendelse fra drivhusgasser mulig (lavere tryk = færre kollisioner).

    William Happer siger om drivhusgasmolekylets vibration efter at have opfanget en foton, at dets svingning er så ekstremt langsom, at energien mistes ved talrige kollisioner med de ’almindelige’ molekyler. Det danner derfor ikke en ny foton – molekyletætheden er for høj.

  5. Peter+Stilbs

    Lennart #2 – jag har sett många konstiga kommentarer i detta sammanhang under åren baserade på detta med ”allmänna gaslagen” ( pV=nRT ) – där p är trycket och T är absoluta temperaturen. Man tycks tro att det också i atmosfären därför är ett direkt samband mellan tryck (p) och absolut temperatur (T) .

    Så bra – då skulle temperaturen gå mot 0 Kelvin när man stiger upp i atmosfären… – och man behöver inga kylskåp – det är bara att ändra trycket på något

  6. tty

    Det finns flera fel i ”tankeexperimentet”, t ex att Jordens albedo skulle vara detsamma utan växthusgaser. Det skulle det garanterat inte vara eftersom det vare sig skulle finnas moln, hav eller växtlighet.

  7. Lars Kamél

    Närvaron av en atmosfär gör att jordytan har möjligheten till fler avkylningsmekanismer än strålning: Konvektion, avdunstning, ledning. Ledning är så ineffektivt att det kan försummas. Konvektion och avdunstning är viktiga för att föra värme till så hög höjd att strålning blir den effektivaste avkylningsmekanismen.
    Utan en atmosfär, skulle albedot vara ett helt annat, eftersom det inte skulle finnas moln, flytande vatten eller växter. Tankeexperimentet är bara ett tankeexperiment, om något som inte kan inträffa i verkligheten.

  8. Lasse

    Tack-håller med #1 🙂
    Funderar på hur det skulle gå att göra detta mer lättbegripligt eller experimentellt vederlagt.
    Tittade på denna; http://ocean.dmi.dk/arctic/meant80n.php
    Vinterns solskugga ger en medeltemp som är 243 K är det relevant i denna diskussion?
    Olika planeter borde väl ha olika yttemperaturer-vad säger det?
    Eller man kanske kan göra som Lennart B föreslår-steka köttbullar 🙂

  9. Bjarne Bisballe

    Om man i tankeeksperimentet har en klode med drivhusgasfri atmosfære eller om man har en klode helt uden atmosfære vil ikke gøre nogen forskel, fordi den eneste måde indstrålet Solenergi kan ende ude i verdensrummet på, er ved infrarød udstråling. Det kan i begge tilfælde kun ske fra jordoverfladen/markytan

  10. tty

    ”Hur stor del av den senare värmetransporten sker genom strålning, ledning, konvektion respektive processer som vattenavdunstning och därav föranledd transport av så kallad latent värme? ”

    Ledning kan man i praktiken bortse från, gaser har mycket dålig ledningsförmåga. Avdunstning/kondensation är i praktiken en del av och förstärker konvektionen, och t o m IPCC medger att konvektion är en viktigare mekanism för värmetransport från ytan än strålning, även om man måste leta för att hitta det.

    ”Inom nutida klimatologisk forskning godtar man emellertid inte antagandet att troposfärens temperatur avtar linjärt från jordytan ända upp till tropopausen.”

    Det är den inte riktigt heller. Dock inte av de skäl Manabe anger, utan för att mängden vattenånga minskar med höjden, detta sänker luftens värmekapacitet och förskjuter lapse-raten närmare torradiabaten. Den strikt linjära temperturminskningen i ”Standard Atmosphere” är en förenkling. Detta kan ses på denna site:

    http://climatemodels.uchicago.edu/modtran/

    Temperaturprofilen ligger till höger, byt atmosfärfall till vänster (”Tropical Atmosphere” är default) och se hur profilen varierar med tid och plats. Man ser att lapse-raten i stort sett i alla fall ökar med höjden.

    Den höjd är radiativa effekter tar överhanden är inte 8 km utan tropopausen som ligger på 11 km i ”standardatmosfären”, men varierar från ca 9 till ca 18 km beroende på breddgrad och årstid. Högst i tropikerna och lägst i Arktis.

    Sedan kompliceras bilden av ”medelutstrålningshöjden” av att vi hela tiden har att röra oss med två utstrålningsmekanismer. Gaser i atmosfären som strålar inom diskreta band (även icke-växthusgaser strålar, dock inte i IR-området) och markytan och moln som är (i stort sett) svartkroppsstrålare med utstrålning inom hela spektrat.

    I klar luft har vi, något förenklat, alltså markens svartkroppsstrålning i ”fönstret” och växthusgasernas utstrålning inom deras respektive band. Den första delen styrs av marktemperaturen och den andra av temperaturen på respektive utstrålningshöjder.

    När det finns moln flyttas i princip markytan strålningsmässigt upp till molnöversidan och med den temperatur som råder där (plus att en stor del av solstrålningen då reflekteras från molnet och aldrig når marken).

    Observera att den gravitotermala effekten enbart beror av atmosfärens värmekapacitet och gravitationsfältets styrka, mängden växthusgaser spelar ingen roll för effektens storlek. Däremot kan den ”parallellförflytta” temperaturprofilen mot högre värden.

    En annan intressant slutsats är att om Jorden hade en lika tät atmosfär som Venus skulle det vara ungefär lika varmt här. Venus gravitationsfält är nästan detsamma som Jordens, men atmosfären är ca 60 km djupare. 60 x 6,5 = 390 grader.

    #3

    ”Om man istället räknar med lokal strålningsjämvikt, vilket är det fysikaliskt korrekta, så blir polerna kallare och ekvatorn varmare. P.g.a. olineariteten i Stefan-Boltzmanns lag (temperaturen upphöjt till 4) kommer strålningsbalans inträffa vid en betydligt lägre medeltemperatur än -18C. ”

    Ett mycket viktigt påpekande! Denna effekt minskas dock av att atmosfären och havet transporterar värme från ekvatorn till högre breddgrader innan den strålas ut.

  11. tty

    #9

    ”Om man i tankeeksperimentet har en klode med drivhusgasfri atmosfære eller om man har en klode helt uden atmosfære vil ikke gøre nogen forskel, fordi den eneste måde indstrålet Solenergi kan ende ude i verdensrummet på, er ved infrarød udstråling. Det kan i begge tilfælde kun ske fra jordoverfladen/markytan”

    Lapse-raten i en sådan atmosfär skulle dock vara densamma, men den skulle troligen vara mera instabil eftersom värmeöverföringen mellan markytan och atmosfären skulle vara liten. Det förutsätter också att det inte finns några partiklar (stoft, vulkanaska) i atmosfären eftersom de fungerar som svartkroppstrålare. Dessutom strålar faktiskt även icke-växthusgaser. Dels svagt och bredbandigt inom IR-området i samman med molekylkollisioner men framförallt utanför det. Syre har t ex emissionslinjer i mikrovågsområdet.

    Men det är svårt att tänka sig en atmosfär utan växthusgaser. På mycket kalla planeter finns visserligen mycket litet CO2 i atmosfären, men där finns troligen CH4 och andra kolväten istället. Möjligen skulle man kunna tänka sig en atmosfär av enbart helium på någon extremt kall planet någonstans.

  12. Bjarne Bisballe

    Med udgangspunkt i den gravitotermale effekt er temperaturen i atmosfæren lavere i højderne sammenlignet med temperaturen ved jordoverfladen/markytan og da jordens klimasystem skal afgive lige så meget energi i form af strålingsenergi til verdenrummet, som det modtager fra Solen, vil det som et gennemsnit ske fra en højde i atmosfæren, som energimæssigt svarer til den energi som klimasystemet løbende modtager fra Solen, nemlig 240 W/m2 (Watt pr kvadratmeter jordoverflade).

    Hvis temperaturen ved jordoverfladen er plus 15C (svarende til 390 W/2), sker udstrålingen fra 5076 meters højde, hvor temperturen er minus 18C, netop svarende til 240 W/m2, idet der er et 33 graders fald i temperatur fra jordoverfladen til 5076 meters højde (6,5 grader pr kilometer). De 150 W/m2 (ud af de 390 W/m2 der løbende tilføres atmosfæren fra jordoverfladen) som ikke udstråles, men i stedet returnerer til jordoverfladen, holder atmosfæren og jordoverfladen opvarmet (med 33 grader).

    Ved en fordobling af CO2 i atmosfæren vil disse 150 W/m2 stige til 153,7 W/m2 og jordoverfladens udstråling til 393,7 W/m2. Det vil give en temperaturstigning i jordoverfladen og troposfæren på ca 0,7 gr og en opflytning af udstrålingshøjden på ca 100 meter.

  13. Torbjörn

    #9 Bjarne
    Det är skillnad i instrålning på en planet med atmosfär än en planet utan atmosfär på grund av refraktion

  14. Om jag förstått Göstas argumentation så är den alltså ungefär följande: Den av IPCC framräknade effekten av jordens atmosfär är 32 grader C. Denna effekt har Maxwell kommit fram till med klassisk statistik och med hänsynstagande till konvektionens inflytande. Däri finns gravitationskonstanten g med. Alltså är lapsraten en gravitationell effekt.

    På den andra sidan finns strålningsfysik och kvantmekanik: Enligt dessa så finns det en påvisbar uppvärmningseffekt av växthusgaser som har en förmåga att absorbera och emittera strålning. Den förmågan är som störst vid små halter av t.ex. CO2, mellan 0 och 300 ppm. Därefter avtar effekten. Effekten följer en logaritmisk kurva. Alltså MÅSTE en DEL av lapsratens utseende i jordens atmosfär vara en konsekvens av existensen av växthusgaser.

    Jag får inte de båda delarna av välkänd fysik att gå ihop. Någon har räknat fel någonstans 🙂

  15. Peter+Stilbs

    Ingemar #14 – vad jag ser – och som flera kommenterat – är lapse rate gravititionellt bestämd – och beskriver energins oförstörbarhet i ett isolerat (adiabatiskt) system (kinetisk (värme) – och potentiell). Däremot är det en annan fråga HUR energin kom dit.

    Lokal energiöverföring via strålningsutbyte eller molekylkollisioner sker på mycket kort tid – storleksordning nanosekunder

  16. OT Den här dagen 1984 var det varmt
    i Norrbotten. Vi var på en friidrottstävling i Torneå. Där var det 27 grader i skuggan. Bland det varmaste i Europa den dagen. Ett väldigt bra väder för friidrott. Johan Rockström hade nog hellre sagt att det var ett dåligt väder för planeten.

  17. Adepten

    Ja, nu tycker jag att ingenting stämmer längre!
    Nu är det dags för Göran Åkesson och Leif Åsbrink att kommentera om det är Växthusgasteorin eller Maxwells gravitotermiska teori eller hur de kompletterar varandra. Skillnaden på 33 grader kan förklaras med växthusgasteorin. Om det ska förklaras med den gravitotermiska teorin skulle det strida mot energiprinciperna.

    Maxwells gravitotermiska teori påminner också om Nikolov-Zellers teori om planeters yttemperaturer bara är en funktion av absorberat solinstrålning och atmosfärstryck.

    Göran Åkesson anser att Professor Happer misstar sig beträffande växthusgasers effekt. Den kan inte återge atmosfärens temperaturprofil samt att Jordytan värms upp endast av solinstrålningen enligt hans uträkning. Han anger ingen gravitotermisk effekt som skulle värma upp Jordytan. Görans teori är intressant om den kan ledas i bevis?

    Leif Åsbrink anger växthuseffekten som orsak till skillnaden på 33 grader. Det är allmänt accepterat.

  18. Gösta Pettersson

    #2 Lennart Bengtsson
    ”Som lätt inses från vertikalkomponenten av Navier-Stokes rörelseekvation …”

    Som jag antydde i gårdagens inlägg finns det många sätt att komma fram till de av Loschmidt härledda sambanden för den troposfäriska temperaturgradienten, t ex det du nämner. Men observera att även en härledning via Navier-Stokes ekvation framställer gradienten som en gravitationell effekt med gravitationskonstanten som en karakteriserande parameter.

  19. Gösta Pettersson

    #3 JonasW
    ”beräkningen av jordens jämviktstemperatur utan atmosfär (-18C) bygger på att jorden är jämnvarm. … Om man istället räknar med …”
    #6 tty
    ”Det finns flera fel i ”tankeexperimentet”,

    Håller med om att det kan finnas starka invändningar mot tankeexpeerimentets utformning. Men Inlägget handlar om den kritik som riktats mot IPCC:s förstahandsbevis för existensen av en signifkant (32-gradig) naturlig växthuseffekt. Det är därför jag håller mig till de förutsättningar som t ex SMHI anger för tankeexperimentet.

  20. GoranA

    Först som sist vill jag tacka Gösta P för två intressanta inlägg.

    Om jag har förstått det rätt så beräknas den totala naturliga växthuseffekten till ungefär 33°C. Denna temperatur får fram genom att ta differensen mellan en beräknad temperatur från temperaturdata från termometrar och en beräknad temperatur från den medeleffekt som når Jorden.

    I det första fallet används ett snurrande jordklot vars yta till stor del består av hav is och land och en atmosfär som till en liten del består av IR-aktiva gaser. I det andra fallet räknar man på ett jordklot utan hav, is och atmosfär

    Trots de uppräknade skillnaderna så antas hela temperaturskillnaden uppkomma p g a de IR-aktiva gaserna i atmosfären.

    Jag kan tycka att borde försöka beräkna den temperatur som uppstår i på Jorden i det fallet man bara ta bort de IR-aktiva gaserna från atmosfären jämfört med en vår naturliga Jord.

    Enligt ovan så skulle en sådan jord bara kunna avge energi från Jordytan via strålning men stämmer det.

    Jag tänker mig att solens instrålning leder att olika delat av jordytan värms olika vilket gör att vi får olika uppvärmning av luften ovanför via ledning vilket leder till konvektion. Konvektionen leder till tryckskillnader i atmosfären som ger upphov till vindar som mångfalt ökar ledningen av värme från Jordytan.

    Vi skulle få en atmosfär som värms från Jordytan. Denna atmosfär skulle få svårt att bli av med sin energi om den inte effektivt kan stråla ut i IR. Vi skulle enligt mitt scenarie få en temperatur som överstiger -18°C på Jorden.

    Den temperatur som en atmosfär skulle uppnå utan växthusgaser tror jag är betydligt högre än den man tar för given när man bara räknar på en stenplanet och vad är då en sådan beräkning värd.

  21. Torbjörn

    Tycker hela diskussionen spårar ut lite då alla tycks mena olika saker.
    Det finns säkert någon som kan förklara atmosfären och dess sammansättning bättre än mig, men alla gaser i atmosfären är inte växthusgaser, så man kan inte säga att det inte finns någon atmosfär om man tar bort växthusgaserna.
    Om man mot förmodan skulle ta bort alla växthusgaser, hur skulle jordens temperatur förändras?

  22. Bjarne Bisballe

    #20 GoranA
    Her William Happers forklaring:

    ”From Earth’s surface the energy emission to space is exactly the same in W/m2 as received from the Sun as the emission from it is a function of the temperature – and as the atmosphere does not stop any IR-emission, the surface temperature must be what is dictatet by the beams from the Sun. – In daytime conduction cools the surface (and warms the atmosphere) in the nighttime the conduction warms the surface (and cools the atmosphere).

    The atmosphere cannot store heat from one day to the next. If it could, the surface would be warmer and warmer an warmer and warmer….. The only way the atmospher can get rid of it’s enegy is by warming the surface as the direct road to space is closed”

  23. Håkan Bergman

    Torbjörn #21
    ”Om man mot förmodan skulle ta bort alla växthusgaser, hur skulle jordens temperatur förändras?”
    Det finns dom som jobbar för att tömma atmosfären på åtminstone CO₂, skulle dom mot all förmodan lyckas kommer vi inte att märka nån skillnad eftersom vi svultit ihjäl innan det gått så långt.

  24. Gösta Pettersson

    #14 Ingemar Nordin
    ”Någon har räknat fel någonstans”

    Det rör sig nog inte om felräkningar, utan om vilka förutsättningar man antar gälla vid beräkningarna.

    Strålningsfysiker utgår normalt från att det uppnås en radiativ jämvikt (och kommer då fram till en logaritmisk kurva för temperaturgradiententen, i strid med gjorda observationer).

    Den Maxwellska skolan säger att någon radiativ jämvikt aldrig hinner uppnås, eftersom konvektion ger snabbare värmetransport än strålning och medför att temperaturgradienten blir konstant (linjärt avtagande temperatur med ökad höjd) i enlighet med Maxwells ”konvektionella termiska jämvikt”, Loschmidts ekvation, samt gjorda observationer.

    Beträffande inläggets rubricerade tema behöver man egentligen inte bekymra sig om strålningseffekter. GHE-opponenternas av mig beskrivna kritik baserar sig helt på konsekvenserna av att det existerar en temperaturgradient av gravitationellt ursprung.

  25. Gösta P #24,

    Min fundering: Egentligen är ju denna debatt lite vid sidan om huvudspåret i ”klimatdebatten” idag. Du vill tillbakavisa och förklara temperaturgradienten genom att hänvisa till att den lika gärna kan fås fram genom statistisk mekanik a la Maxwell. Men såvida du inte menar att strålningsfysiken och kvantmekaniken är helt fel (likt Claes Jonsson) så återstår det ju att beskriva vilken inverkan det fortsatta utsläppen av växthusgaser har för jordens temperatur.

    Jag kan inte se att du motbevisat att växthusgaserna har en värmande effekt vid jordytan. Du har inte bevisat att växthusgaserna har noll effekt (det handlar om enstaka grader som jag ser det). Däremot känns din argumentation mot att en del växthusgastroendes (typ James Hansen) argument att växthusgaserna skulle kunna förklara hela temperaturgradienten övertygande.

    Våra små CO2-utsläpp påverkar ju knappast atmosfärens massa utan den är rätt konstant över årmiljonerna.

    Det jag menar med att ”räkna fel” är väl snarast att jag ser en motsättning – jag gillar sådana eftersom de brukar driva vetenskapen framåt.

    Skeptikernas position brukar vara att växthusgasernas uppvärmande effekt är liten, och minskar logaritmiskt med tiden. Medan alarmisternas position är att växthusgaserna betyder allt – åtminstone sedan 1950. Den avtagande effekten av CO2 tycks man mer eller mindre ignorera när man går ut med sina katastrofscenarier.

  26. Paul Håkansson

    OT

    Nu får danska elproducenter betala för att bli av med sin vindkraftsel 7 öre. I södra Sverige är det gratis för konsumenterna och övriga Sverige kostar elen 5 öre.

  27. GoranA

    #22 Bjarne
    In daytime conduction cools the surface (and warms the atmosphere) in the nighttime the conduction warms the surface (and cools the atmosphere).
    Är dessa processer likvärdiga?
    Vad händer om du introducerar hav på 70 % av yta?

    ””The atmosphere cannot store heat from one day to the next.””
    Varför inte?
    Är det helt oberoende av dygnslängden, timmar till år?

    If it could, the surface would be warmer and warmer an warmer and warmer….. The only way the atmospher can get rid of it’s enegy is by warming the surface as the direct road to space is closed””

    Vad händer om du introducerar en växthusgas (H2O) får du en värmande eller en kylande effekt på en sådan atmosfär?

  28. Gösta Pettersson

    #17 Adepten
    ”Skillnaden på 33 grader kan förklaras med växthusgasteorin. Om det ska förklaras med den gravitotermiska teorin skulle det strida mot energiprinciperna.”

    1. GHE-opponenternas kritik baserar sig sammanfattat och något förenklat på följande argument:

    Den troposfäriska temperaturgradienten i vertikal riktning är en gravatationseffekt. Den sträcker sig enligt meteorologernas standardbeskrivning från jordytan upp till tropopausen.
    Därav följer med matematisk nödvändighet att temperaturskillnaden mellan vilka som helst två höjdmässigt skilda troposfäriska skikt återspeglar en gravitationseffekt. Samt som specialfall att den 32-gradiga temperaturskillnaden mellan utstrålningsskiktet (-18 °C) och den marknära luften/jordytan (14 °C) representerar en gravitationseffekt.

    Det finns ingenting i resonemanget som strider mot några energiprinciper. Argumentet pekar enbart på konsekverna av att troposfären uppvisar en temperaturgradient som är av gravitationellt ursprung.

    2. ”Skillnaden på 33 grader kan förklaras med växthusgasteorin. ”
    Det finns inte en, utan snarare ett tjugotal hypoteser om hur växthuseffekter kan uppkomma. Jag tillåter mig att betrakta Wanabes förra året nobelprisade modell för växthuseffekten som den som för närvarande betraktas som tillförlitligast av GHE-proponenter.

    Och Wanabes modell ger inget som helst stöd för att den ca 32-gradiga temperaturskilnaden mellan utstrålningsskiktets -18 °C och jordytan representerar en växthuseffekt. Tvärtom framställer modellen sagda temperaturskillnad som en gravitationell effekt (den faller inom det höjdmässiga område där temperaturerna modellmässigt anses vara bestämda av den ”konvektiva jämvikten”).

  29. Bjarne Bisballe

    #27 GoranA
    Vad händer om du introducerar hav på 70 % av yta?
    Det vil være det samme. Argumentet er, at hvis ikke man er tilbage til start hver morgen, ”surface would be warmer and warmer an warmer and warmer…..”
    Processerne er ligeværdige.

    Är det helt oberoende av dygnslängden, timmar till år?
    Det vil være ’år’, da der er lidt variation fra dygn til dygn, men udlignet over et år.

    Vad händer om du introducerar en växthusgas (H2O) får du en värmande eller en kylande effekt på en sådan atmosfär?
    Den vil være varmende på atmosfæren og i første omgang afkølende på markytan, som så senere vil få den mistede energi tilbage og med tiden udsende endnu mere energi. Der vil så senere indfinde sig en ligevægt med både højere atmosfæretemperatur og markyta-temperatur samt en løbende vedligeholdt energimængde i atmosfæren.

  30. Bim

    Göran A # 27
    Bra fråga.
    Jag kan ju inget om gasernas klimateffekter och strålningsförhållandena i atmosfären.
    Men vissa enkla frågor uppstår ju när det verkar vara så komplicerat att de som kan inte är överens.

    Vad jag dock tror mig fattat är att CO2, en knapp halv tusendel av atmosfären, skulle vara så kraftfull växthusgas att vi snart har fördärvat hela vår elförsörjning. och skrämt halv ihjäl våra barnbarn.
    Därför har jag ibland undrat om fenomenet i varma öknar . Nära 50 grader på dagen och mycket kalla på natten.
    Jag har då helt ovetenskapligt funderat på om det är bristen på vattenånga som är orsaken. Vattenångan är ju ungefär 20 % av atmosfärens gaser. Vad jag förstår så är CO2 ganska jämnt fördelat över jordytan men i detta fall verkar den vara en dålig växthusgas som inte kan hålla kvar värmen i öknar.

    Har försökt följa Era teorier om strålningens fysik och börjar undra. Vet vi alls hur farlig CO2 är.

    Jag skulle gärna vilja höra tty,s åsikt .
    Har vi överhuvud taget en ”klimatkris”? Har fått en känsla av att han är ytterst påläst och kunnig.

    Och en eller tvågraders varmare jord plus lite mer CO2 skulle ju ge bättre växtlighet som genererar mindre svält ,billigare mat ,
    Vad är problemet med att vissa glaciärer smälter? Grönland och Antarktis lär nog klara det som de gjort andra värmeperioder.

  31. Munin

    # 24
    ”Den Maxwellska skolan säger att någon radiativ jämvikt aldrig hinner uppnås, eftersom konvektion ger snabbare värmetransport än strålning och medför att temperaturgradienten blir konstant (linjärt avtagande temperatur med ökad höjd) i enlighet med Maxwells ”konvektionella termiska jämvikt”, Loschmidts ekvation, samt gjorda observationer.”

    Jag drar slutsatsen att konvektion är huvudregulatorn för klimatet och att detta eliminerar koldioxid som ett klimathot. Konvektionen (och dess energiflöden) korrigerar hela tiden upp även för den delen. Det gör att klimatforskningen borde ändra fokus, bort från koldioxiden, och i stället lägga energin på vatten och dess roll i klimatets energiflöden.

  32. Gösta Pettersson

    #25 Ingemar Nordin
    ”Du har inte bevisat att växthusgaserna har noll effekt”

    Nej det har jag definitivt inte. Det har heller inte varit min ambition, eftersom jag är övertygad om att det existerar växthuseffekter. För mig är den kritiska frågan om växthuseffekter är av sådan styrka att de signifikant påverkar jordytans temperatur.

    Och då har jag valt att som delsvar lyfta fram GHE-opponenternas ifrågasättande av IPCC:s påstående att jorden uppvisar en 32-gradig naturlig växthuseffekt. Och deras slutsats att det snarare är så att den naturliga växthuseffekten är av insignifikant storlek.

    Och är den naturliga växthuseffekten av obetydlig storlek, så lär den förstärkta växthuseffekten vara ännu obetydligare. Det inser även klimatalarmister. Det är väl därför det brukar bli så inflammerade debatter varje gång den gravito-termala effekten tas upp till behandling.

  33. Thomas P

    Att koldioxiden har en logaritmisk inverkan har funnits med i beräkningarna sedan Arrhenius. Konvektion, det GP kallar ”gravito-termal effekt”, har tagits med i alla fall sen Hulburts artikel 1931 osv. De flesta inläggen här baseras tyvärr på att ni aldrig orkat sätta er in i hur klimatmodeller fungerar.

    Skulle GP verkligen försöka räkna på saken misstänker jag att han, liksom Happer, skulle komma fram till ungefär samma resultat för klimatkänsligheten som alla andra.

  34. För att förstå ”växthuseffekten” i ljuset av vad Gösta anför måste man vara pedant. Jag gör ett försök att beskriva fysiken pedantiskt.

    Först Göstas referat av vad IPCC säger:
    ”En atmosfär utan växthusgaser kan inte stråla ut värme mot rymden, utan värmeutstrålningen måste ske från jordytan. I en tänkt frånvaro av atmosfäriska växthusgaser måste alltså Jorden strålningsmässigt bära sig åt som om den helt hade saknat atmosfär och antaga yttemperaturen -18 °C. Den 32 °C högre yttemperatur som Jorden med sin växthusgasinnehållande atmosfär i realiteten befunnits uppvisa måste därför representera en växthuseffekt.”

    För det första är ordet växthuseffekt inte så väl valt. Jag fortsätter att använda det, men menar med ordet ”effekt av att atmosfären innehåller molekyler med absorption i det infraröda området.” Likaså använder jag ordet växthusgas men menar med det: ”molekyler med absorption i det infraröda området”

    Göstas invändning :”Det är inte någon hållbar slutsats. Må vara att närvaron av växthusgaser måhända krävs för att atmosfären ska kunna ge någon förhöjning av jordens yttemperatur överhuvud taget. Men därav följer inte att den kalkylerade 32-gradiga atmosfärseffekten till hundra procent måste anses representera en växthuseffekt. Ytterligare argument krävs för att avgöra hur mycket av den beräknade 32-gradiga temperaturförhöjningen som kan förmodas hänföra sig till en strålningsrelaterad växthuseffekt och hur mycket som kan förmodas hänföra sig till de temperaturpåverkande sekundäreffekter som måste uppkomna i en atmosfär som innehåller växthusgaser.”

    Här är det fråga om semantik och definitioner, inte fysik. Jag kan formulera om Göstas invändning: ”Må vara att vi vet att växthusgaser förorsakar en viss uppvärmning. Uppvärmningen kan ändra luftfuktigheten och därmed ändra storleken på växthuseffekten. En återkopplingsmekanism – men det är fortfarande växthuseffekt. Därtill kommer att jorden förändras. En jord utan växthusgaser skulle inte kunna ha några moln eller någon växtlighet och skulle därför ha ett albedo som månen, omkring 0,12. Utan vatten, mycket kallare poler och varmare runt ekvatorn. Större skillnad mellan dag och natt.

    Den 32-gradiga växthuseffekten är ett något missledande sätt att beskriva jordens strålningsbalans i termer av temperaturer. Det fungerar när man använder det för att diskutera små förändringar i strålningsbalansen men man måste hela tiden hålla i minnet att de 32 graderna är observerad ”medeltemperatur” jämfört med temperaturen hos jorden i ett tankeexperiment där hela ytan året runt, dygnet runt, har samma temperatur och där ingen växthusgas finns i eventuell atmosfär.

    Jag har satt ”medeltemperatur” inom citattecken, därför att jag inte vet om det verkligen är medeltemperatur i bokstavlig mening eller om det är fjärde roten ur medelvärdet av T4, temperaturen upphöjt till fyra, som man använder det som om det skulle vara.

    Nu spelar de 32 graderna ingen roll. Det enda som är av intresse är hur störningar i atmosfären kan tänkas ändra växthuseffekten. Växthuseffektens absoluta storlek saknar helt mening. Det är en hjälpstorhet som används för att försöka bibringa allmänheten en ungefärlig uppfattning om betydelsen av växthusgaser i atmosfären. Denna etablerade praxis är måhända olycklig, det hade nog varit bättre att ange växthuseffekten till X grader och säga att hur en jord utan växthusgaser skulle se ut kan vi inte veta. Det fiffiga är att X försvinner när man försöker uppskatta förändringar av växthuseffekten.

    Gösta skriver sedan: ”Det diskuterade tankeexperimentet kan förvisso tas som belägg för att det föreligger en 32-gradig atmosfärseffekt. Men vill man i likhet med IPCC (samt SMHI och GHE-proponenter i allmänhet) hävda att detta är en växthuseffekt, så krävs det vetenskapligt betydligt mera övertygande argument än det trosvissa påståendet att så är fallet.” Tvärtom. IPCC m.fl. menar att detta utgör själva definitionen på växthuseffekten. Den är godtycklig i den meningen att deras referensjord kommer från ett orealistikt tankeexperiment.

    Bättre hade varit om man i stället hade räknat ut är fjärde roten ur medelvärdet över tid (1 år) och hela jordytan av T4, temperaturen upphöjt till fyra. Delat med hela jordens yta blir det då 288K. Lite krångligare att förstå som första steg i en förklaring, men man undviker vad jag förstår de problem som Gösta hakar upp sig på. Sedan kan man tillägga: En jord utan växthusgaser skulle vara annorlunda. Vi ansätter ett årsmedelvärde på strålningen som motsvarar en svartkropp med temperaturen X vars värde inte påverkar hur växthueffekten förändras av små förändringar av atmosfärens sammansättning. (Att det är brukligt att ange X till 32 grader saknar fysikalisk mening.)

    Nästa steg i resonemanget:
    ”Dessa utstrålningsskikt kommer jag att tankeexperimentellt medelvärdesutjämna och globalt generalisera till ett utstrålningsskikt, med acceptans av GHE-proponenternas förmodande att skiktet uppvisar den balansskapande temperaturen -18 °C.

    Alla lär vara överens om att full förståelse av energiflödet från solen till Jorden och därifrån vidare ut i rymden kräver beaktande av värmetransporten från jordytan till utstrålningsskiktet.”

    Här går det fel. Jag återkommer i nästa kommentar.

  35. GoranA

    #29 Bjarne
    Du svarar inte på en av mina frågor angående:
    ”The atmosphere cannot store heat from one day to the next. ”
    Jag frågar: Varför inte?
    Som jag ser det har atmosfären massa och volym, vilket andra fall innebär att värme kan lagras.

  36. JonasW

    #5 Peter

    Håller med. Dessutom finns det obekant till – n. D.v.s. tätheten eller densiteten.

    Man har tre obekanta p, T och n. Det går inte att lösa detta med statistisk mekanik.

    Som jag skrivit tidigare så är nog den bästa teoretiska modellen virialteoremet. Då behöver man inte göra några antaganden om adiabatisk expansion o.s.v.
    Virialteoremet kopplar ihop potentiell och kinetisk energi i ett gravitationsfält.

    #14 Ingemar

    Maxwell har inte visat att atmosfären höjer temperaturen med 32 grader. De 32 graderna kommer från att man räknar strålningsbalans mellan solen och en jämnvarm jord (vilket den inte är).

    Sen håller jag med om att det är lite vid sidan om, men det är ända en intressant diskussion tycker jag.

    —————-

    Gravitation kan inte öka jordens temperatur eftersom den inte kan utföra ett nettoarbete. Om gravitationen ”värmer” jorden så kommer gravitationen att minska med tiden. Det gör den inte.

  37. Torbjörn

    #35 JonasW
    Hur menar du att gravitationen inte skulle kunna utföra ett nettoarbete, det finns ju andra krafter som arbetar utåt/uppåt?

  38. JonasW

    #36 Torbjörn

    Om man kastar en boll upp i luften så kommer gravitationen att dra ner bollen igen. Gravitationen utför inget nettoarbete när bollen är tillbaka på marken. Den energi som eventuellt frigjordes när bollen slog i marken kommer från den som skickade iväg bollen. Inte från gravitationen. Gravitationsfältet gör inget nettoarbete i detta fall.

    Om gravitationen utför ett nettoarbete så måste energin tas från gravitationsfältet. Det är enda sättet som gravitation kan överföra energi till något annat. Enligt energiprincipen som måste då energin i gravitationsfältet minska, d.v.s. gravitationen avta.

    Om något åker upp så kommer det ned, men energin kommer från det som skickade upp något. Inte från gravitationsfältet.

  39. Detta stämmer inte så bra: ”Alla lär vara överens om att full förståelse av energiflödet från solen till Jorden och därifrån vidare ut i rymden kräver beaktande av värmetransporten från jordytan till utstrålningsskiktet.”

    Avgörande för förståelsen av växthueffekten är kunskapen om att temperaturen avtar med höjden på ett sätt som vi har god kunskap om. Det är inte den teoretiska ”gravito-termala effekten” utan vår experimentellt kända kunskap som är avgörande. Vi vet att ibland uppstår inversioner där temperaturen stiger med höjden (varm fuktig luft kan glida in över kall luft) Inversioner kan täcka mycket stora områden, hundratals mil tvärs över. Det finns även på sydpolen där temperaturen normalt stiger med höjden, åtminstone vintertid. (Där är CO2 kylande!)

    Det är alltså experiment och inte teori som ligger till grund för kunskapen om den vertikala temperaturfördelningen och dess variation med latitud och årstid.

    För att förstå energitransporten från jorden ut i rymden behöver vi inte förstå hur det kommer sig att temperaturen avtar med höjden på det sätt som den faktiskt gör (eller inte gör.) Mekanismerna spelar ingen roll. Latent värme, konvektion, strålning eller något annat spelar ingen roll. En del av värmen i atmosfären kommer från solens infraröda strålning. Dessutom finns partiklar som värms av både synligt och infrarött ljus.

    Avgörande är att det räcker att känna till temperaturen. Det skriver ju Gösta också: ”Men beträffande detta inläggs rubricerade tema är detaljmekanismerna för värmetransporten från jordytan till utstrålningsskiktet av underordnad betydelse. Vad man överordnat kan konstatera är att det finns en värmetransportsom leder till en balanserad situation med ett utstrålningsskikt vars medeltemperatur är sådan att den normalt medför att Jordens energiavgivning blir lika stor som dess energiupptag från solen.”

    Här kommer felet i Göstas resonemang: ”Det är nämligen gradientens lapse rate som bestämmer vilken temperaturskillnad det kommer att föreligga mellan jordytan och utstrålningsskiktet på den höjd det senare hamnar vid balans mellan instrålning och utstrålning av energi till Jorden. När IPCC och GHE-proponenter hävdar att den 32-gradiga atmosfärseffekten utgör en växthuseffekt, så är det ekvivalent med att hävda att den troposfäriska temperaturgradienten helt och hållet återspeglar en växthuseffekt.”

    Sista meningen skall vara: ”så är det ekvivalent med att hävda att höjden för det ekvivalenta utstrålningsskiktet helt och hållet återspeglar en växthuseffekt.” Detta är naturligtvis en sanning med modifikation, se min text ovan. Vad man hävdar är att små förändringar av atmosfärens innehåll av växthusgaser påverkar utstrålningsskiktets höjd. Detta leder till andra ordningens effekter genom att ändrad marktemperatur leder till ändrad luftfuktighet vilket påverkar laps rate en aning. Man benämner den totala effekten av alltsammans ”växthuseffekt” enligt definition ovan.

    Här skriver Gösta ut sin felaktiga tanke nästan i klartext: ”Då måste temperaturskillnaden mellan vilka som helst två höjdmässigt skilda punkter på grafen för våtadiabaten återspegla en gravitationell effekt och inget annat. Varav som specialfall följer att den 32-gradiga temperaturskillnaden mellan utstrålningsskiktets balansskapande -18 °C och jordytans 14 °C återspeglar en gravitationell effekt utan någon som helst anknytning till vissa luftmolekylers speciella förmåga att absorbera infraröd värmestrålning.”
    Fråga: ”Vad bestämmer utstrålningsskiktets höjd”
    Svar: ”Den bestäms av växthusgasernas koncentrationer och absorptionsspektra i infrarött.”

    Slutsatsen: ”Så sammanfattningsvis är den enda hållbara slutsatsen rörande detta inläggs rubricerade tema att tankeexperimentets 32-gradiga atmosfärseffekt måste anses utgöra en gravito-termal effekt. Såväl teoretiska analyser som empiriska observationer ger belägg för att det existerar en gravitationsbetingad 32-gradig atmosfärseffekt som återspeglar luftmolekylernas massa och värmekapacitet, utan någon som helst påvisad kvantitativt signifikant relation till molekylernas spektrala egenskaper och förmåga att absorbera värmestrålning.”

    Sista ledet: ”utan någon som helst påvisad kvantitativt signifikant relation till molekylernas spektrala egenskaper och förmåga att absorbera värmestrålning.” Detta är häpnadsväckande. Gösta tror således att utstrålningsskiktets höjd är oberoende av molekylernas spektrala egenskaper. Har han inte sett spektra av jordens IR-strålning tagna från satellit? Det är uppenbart att strålnigen runt 666 cm-1 är svag på grund av närvaron av CO2. Skulle atmosfären befrias från all CO2 skulle strålningen runt 666 cm-1 i stället komma direkt från marken, dvs det utstrålande skiktets höjd skulle sänkas kraftigt. Därvid skulle naturligtvis marktemperaturen sjunka i enlighet med den gravito-termala effekten.

    Självklart faller också nästa slutsats: ”Det är en väsentlig slutsats, eftersom man från den kan gå ett steg längre och konstatera att den 32-gradiga ’naturliga växthuseffekt’ som IPCC anser föreligga i själva verket tankeexperimentellt har befunnits vara av insignifikant storlek. Det följer av att den 32-gradiga atmosfärseffekten enligt ovan anförda argument helt och hållet kan hänföras till den gravito-termala effekten.”

    Avslutningsvis, Gösta skriver: ”Slutsatsen att det saknas hållbara belägg för existensen av en 32-gradig ’naturlig växthuseffekt’ har på sådana grunder dragits av ett stort antal klimatologiska forskare. Ett tjugotal av dem namnges i en artikel på den skeptiska bloggen The Hockey Schtick.” Dom har alla fel. Det är tragiskt att skeptiker genom att komma med ofysikaliska ”bevis” för att CO2 inte påverkar värmebalansen skapar en befogad misstro mot alla som ifrågasätter tolkningen av de resultat som kommer från IPCC.

    Lennart Bengtsson, Björn Lomborg och Roger A. Pielke kategoriseras av alarmisterna som lika goda kålsupare som det tjugotal klimatologiska ”forskare”. Gösta refererar till. Få röster ropar i öknen, men här är en som jag finner uppfriskande: https://www.youtube.com/watch?v=bfNamRmje-s

    Det behövs röster i det offentliga rummet som kan hjälpa till med att lugna ner klimatpaniken och avstyra alla dyra och ofta kontraproduktiva åtgärder som nu görs i klimatets namn.

    JA! CO2 påverkar energibalansen (dvs temperaturen)
    NEJ! Det ger inte katastrofala effekter på några decenniers sikt.
    Förstörd ekonomi kommer att vara mycket farligare på lång sikt. Även för klimatet.

  40. Lars Kamél

    Hur går det till att komma fram till samma klimatkänslighet som alla andra, när bestämningar och gissningar av klimatkänsligheten varierar mellan mindre en halv grad per fördubbling av koldioxidhalten till över fem grader? Vilka är då ”alla andra”?

  41. tty

    #33

    Har du faktiskt läst Hulburts artikel? Han kom nämligen fram till att effekten inte var logaritmisk.

  42. Gösta Pettersson

    #34 Leif Åsbrink

    Jag har svårt att förstå vad du egentligen vill ha sagt med din kommentar, utöver att det diskuterade tankeexperimentet bygger på förutsättningar som kan ses som tveksamma (jämför kommentarerna #3, #6, #7 och mitt svar i #19).

    Dessutom har skönjer jag en tendens till att du har lika svårt som IPCC att göra en klar distinktion mellan den tankeexperimentellt beräknade atmosfärseffekten och växthuseffekter enligt din egen definition. Hur ska man annars tolka din kommentar rörande den 32-gradiga atmosfärseffekten ”Tvärtom. IPCC m.fl. menar att detta utgör själva definitionen på växthuseffekten.”

    Det förefaller vara så du själv definierar växthuseffekten när du skriver ”Den 32-gradiga växthuseffekten är ett något missledande sätt att beskriva jordens strålningsbalans i termer av temperaturer”?

    Liksom när du skriver ”Nu spelar de 32 graderna ingen roll. … Växthuseffektens absoluta storlek en hjälpstorhet som används för att försöka bibringa allmänheten en ungefärlig uppfattning om betydelsen av växthusgaser i atmosfären.”
    Hur kan de 32 graderna ge en ungefärlig uppfattning om betydelsen av växthusgaser annat än om man förutsätter att de representerar en växthuseffekt?

    Anser man att den 32-gradiga atmosfäreffekten representerar en växthuseffekt , så är det självfallet svårt att ta till sig GHE-opponenternas argumentation att den representerar en gravitationseffekt. Men argumentet är logiskt och kräver endast acceptans av tanken att den troposfäriska temperaturgradienten är av gravitationellt ursprung (se min kommentar #28 punkt 1).

  43. Gösta Pettersson

    #33 Thomas P
    ”Konvektion, det GP kallar ”gravito-termal effekt”

    Nej, konvektion är något helt annat än den gravito-termala effekten. Effekten ger upphov till den för energetiskt slutna system kalkylerade troposfäriska temperaturgradienten. Konvektion ombesörjer enligt Maxwell att gradienten bibehålls även i det reella energetiskt öppna systemet.

    ”Skulle GP verkligen försöka räkna på saken”

    Det är sådana beräkningar som redovisas i mitt inlägg, fast med genvägs-metodik. Den 32-gradiga atmosfärseffekten kan tillfredsställande förklaras som en gravitationseffekt, vilket indirekt säger att den naturliga växthuseffekten är av insignifikant storlek. Och är den naturliga växthuseffekten obetydlig, så lär den förstärkta växthuseffekten vara ännu obetydligare. Det räcker för jag ska tappa lusten att försöka genomföra några klimatkänslighets-kalkyler.

  44. Ivar Andersson

    #33 Thomas P
    Varför behövs 102 eller fler klimatmodeller om man vet hur klimatet fungerar? Det verkar vara mycket som killgissas eftersom det behövs mer än EN klimatmodell.

  45. Sören G

    Var det inte för att man uppmätt Månens medeltemperatur till -18 C som man kom fram till att Jordens medeltemperatur är 32 grader högre p.g.a. att det finns växthusgaser i Jordens atmosfär.
    Men det är ju inte den enda skillnaden mellan Jorden och Månen, vilken befinner sig på samma medelavstånd från Solen. Månen saknar inte bara växthusgaser, den saknar överhuvudtaget gaser, d.v.s. har ingen atmosfär.
    I ett tanke experiment i Fråga Lund hur det skulle vara i botten på Marianergraven ifall man kunde valla in den och tömma den på vatten så räknade man ut att med 11 km djup och 6 C per km så skulle temperaturen där vara 66 grader högre än vid jordytan.
    Ju mer atmosfär över huvudet ju varmare.

  46. Lennart Bengtsson

    jag föreslår att alla intresserade läser Manabes viktiga artikel från 1967. Den förklarar allt väsentligt. Jag förstår helt enkelt inte vitsen med dessa hemmasnickrade hypoteser. Den så kallade 32° diskussion kommer från det faktum att jordens strålningsbalans ger detta värde när jordens balanstemperatur beräknas från Stefan-Bolzmanns ekvation. Se t ex tabell i min bok ”Vad händer med klimatet” Skulle man av någon anledning i ett tankeexperiment eliminera alla växthusgaser från atmosfären skulle resultatet bli en permanent nedisning.
    Det räcker att eliminera all koldioxid. Sedan sköter naturen om resten

  47. Thomas P

    #41 tty Så här skriver Hulburt

    ”Calculation shows that doubling or tripling the amount of the carbon dioxide of the atmosphere increases the average sea level temperature by about 4° and 7°K, respectively; halving or reducing to zero the carbon dioxide decreases the temperature by similar amounts. ”

    Det är inte exakt logaritmiskt, men mer logaritmiskt än linjärt.

  48. Thomas P

    #43 Det *blir* ingen temperaturgradient i ett termiskt isolerat system utan det blir isotermt. Läs på om termodynamikens andra huvudsats!

    Sen frågade jag inte efter något ”indirekt” utan vad du får det till att effekten av en dubblad CO2-halt är. Alternativt kan du skatta vad du menar att jordens temperatur skulle vara om atmosfären helt saknade växthusgaser. Var försiktig med att du här inte också bryter mot termodynamikens första huvudsats.

  49. Bjarne Bisballe

    #35 GoranA
    Hvis atmosfæren på kortere sigt blev varmere, ville også jordoverfladen/markytan blive varmere, og dermed ville den udstråle mere energi til verdensrummet end den modtager, og vil derfor i første omgang afkøles. Ekstravarmen i atmosfæren vil da varme den afkølede jordoverflade op og dermed mister atmosfæren sin ekstravarme.
    Set over et helt år vil der ikke være nogen varmeophobning.

  50. Björn

    En atmosfär utan dynamik är statisk, men den är ju inte det, för vi har ständiga tryckförändringar. Detta är ju egentligen den populariserande ”pumpen”, som vid högtryck, förutom gravitationens egentryck, adderar det tryck som tryckförändringen givit upphov till. Ett förändrat tryck medför enligt allmänna gaslagen en ökad kinetisk energi i de molekyler som utsätts för det förändrade trycket. Ökad kinetisk rörelse är enligt fysiken förenat med värme. Vad innebär egentligen en tryckökning om 10-20 hPa? Gaserna närmast jordytan är de som får den högsta påverkan.

  51. GoranA

    # Bjarne
    Jag förstår nog inte hur du tänker men så här tänker jag.

    Markytan värms genom att det tillförs en viss mängd energi från solen under den ljus delen av ett dygn. Markytan värmer atmosfären genom ledning, detta innebär att markytan under dagen blir kallare än den skulle vara utan atmosfär.

    Under natten när det inte tillförs någon energi från solen svalnar markytan via strålning men värms av den lagrade energi i atmosfären vi ledning.

    Jag tänker mig att omblandningen av atmosfären är effektivare under dagen genom konvektion och vindar vilka jag anser vara svagare på natten än på dagen. Resultatet bör bli att det tar längre tid kyla atmosfären under natten än det tar att värma den under dagen.

    Resultatet blir en högre medeltemperatur för en planet med en atmosfär än en utan.

    Därmed är inte Jordens medeltemp densamma med en atmosfär som utan.

  52. #42 Gösta Pettersson

    Jag gjorde ett försök att utförligt förklara något som Lennart mycket kortfattat säger i #46 Tydligen misslyckat.

    Jag får ett intryck av att du ännu inte läst min kommentar #39.

    Den centrala frågan är ju det du kommit fram till: ”Såväl teoretiska analyser som empiriska observationer ger belägg för att det existerar en gravitationsbetingad 32-gradig atmosfärseffekt som återspeglar luftmolekylernas massa och värmekapacitet, utan någon som helst påvisad kvantitativt signifikant relation till molekylernas spektrala egenskaper och förmåga att absorbera värmestrålning.”

    Detta påstående tolkar jag som att du menar att temperaturen på marken inte skulle påverkas nämvärt om alla koldioxidmolekyler i atmosfären momentant skulle ersättas med kvävemolekyler. Har jag missförstått något?

    Jag vill med bestämdhet påstå att temperaturen skulle sjunka påtagligt inom en timme på platser med molnfri himmel (utom på Antarktis.) Det skulle bero på att utstrålningen mot rymden momentant skulle sjunka med storleksordningen 20 W/m2. Laps rate skulle givetvis vara oförändrat, det är med din terminologi ”utstrålningsskiktets höjd” som minskat kraftigt.

    Det är ett enkelt tankeexperiment. Vad är din åsikt om det?

  53. Gösta Pettersson

    #52 Leif Åsbrink

    Hej Leif! Jag läste redan igår även din kommentar #39, men förhindrades p gr av ett tekniskt missöde från att kunna svara (mina försök att publicera blockerades av bloggens filter).
    Problemet är nu löst, men jag kommer inte att ha tid att kommentera förrän framåt midnatt.
    Så det kommer ett svar.

  54. Gösta Pettersson

    #39 Leif Åsbrink

    Jag startar med ditt nästan inledande yttrande ”Avgörande för förståelsen av växthuseffekten är …”. Jag undrar fortfarande om du kanske menar växthuseffekten som IPCC enligt din menng definierar den, men utgår från att du menar den radiativt relaterade växthuseffekten som du själv, jag och enligt min mening även IPCC definierar den.

    Och då kan jag direkt konstatera att mitt inlägg inte handlar om växthuseffekten eller hur man ska förstå den. Inlägget handlar om den gravito-termala effekten och konsekvenserna av det på grund av denna effekt uppkommer en troposfärisk temperaturgradient av gravitationellt ursprung.

    Så när du några stycken längre ner säger ”Här kommer felet i Göstas resonemang:” kan jag inte finna att du påvisar några fel i min argumentering. Det enda jag kan läsa mig till är din frustration över att jag inte beaktar skeenden som du anser vara väsentliga för förståelsen av växthuseffekters uppkomst. Detsamma gäller alla efterföljande stycken där du anser mig resonera felaktigt.

    Det du har att säga om t ex växthusgasers effekt på strålningskiktets höjd kan säkert vara av intresse att framföra i en diskussion av växthuseffekter, men saknar relevans för vad jag säger i mitt inlägg. De av mig refererade argumenten inkluderar inte några förutsättningar rörande växthuseffekters existens eller styrka. Det är enbart på basis av beläggen för att det existerar en troposfärisk temperaturgradient av gravitationellt urspung som man kommer fram till att den 32-gradiga atmosfärseffekten tillfredsställande kan förklaras som en gravitationseffekt.

    Du säger beträffande det tjugotal forskare som enligt en referens i mitt inlägg framfört kritik mot IPCC:s påstående att jorden uppvisar en 32-gradig naturlig växthuseffekt: ”Dom har alla fel.” Det är ett svagt argument, som tillåter mig upprepa min för styvt ett år framförda förmodan att du kanske inte förstått deras resonemang.

    Vidare ”Det är tragiskt att skeptiker genom att komma med ofysikaliska ”bevis” för att CO2 inte påverkar värmebalansen skapar en befogad misstro mot alla som ifrågasätter tolkningen av de resultat som kommer från IPCC.”
    Några sådana skeptiker känner jag inte till. Ovanstående tjugotal forskare har grundat sina slutsatser på erkänt seriösa fysikaliska teorier och samband, använda inom meteorologin för deras goda beskrivning av empiriska observationer. Och mig veterligt har ingen av dom hävdat att dom ”bevisat” att CO2 inte kan påverka strålningsbalansen.

    För avslutningsvis kan jag citera dig i ett avseende där vi (och förhoppningsvis alla) är helt överens: ”JA! CO2 påverkar energibalansen (dvs temperaturen)”.
    Som jag påpekar i min kommentar #32 är i varje fall jag helt övertygad om att växthuseffekter existerar (kan uppkomma enligt flera realistiska mekanismer). Men den för mig intressanta frågan är hur starka effekterna är. Och i det avseendet är det stor skillnad om det diskuterade tankeexperimentet ger belägg för existensen av en 32-gradig naturlig växthuseffekt (IPCC:s tolkning) eller enligt i mitt inlägg anförda resonemang snarare ger belägg för att den naturliga växthuseffekten är av insignifikant storlek.

  55. Gösta Pettersson

    #52 Leif Åsbrink
    ”Jag gjorde ett försök att utförligt förklara något som Lennart mycket kortfattat säger i #46”

    Lennart Bengtsson rekommenderade alla att läsa Manabes artikel 1967. Det har jag inget emot, eftersom jag i mitt inlägg själv ger referens till artikeln. Jag förmodar att såväl du som Lennart Bengtsson vill informera om att artikeln ger en inom modern klimatologi allmänt accepterad beskrivning av hur man anser växthuseffekten uppkomma. Det har självfallet heller inget att invända mot, mer än att mitt inlägg inte syftar till att belysa växthuseffekters uppkomst.
    Vad som fått mig att uppmärksamma Manabe är att hans modellberäkningar (för 300 ppm CO2) framställer den ca 32-gradiga temperaturskillnaden mellan utstrålningsskiktet och jordytan som en gravitationseffekt. Inte ens GHE-proponenternas modellmässiga flaggskepp ger något stöd åt IPCC:s påstående att det existerar en 32-gradig ”naturlig växthuseffekt”.

    ”Detta påstående tolkar jag som att du menar att temperaturen på marken inte skulle påverkas nämvärt om alla koldioxidmolekyler i atmosfären momentant skulle ersättas med kvävemolekyler. Har jag missförstått något?”

    Ja, att jag hyser någon mening i det avseendet.

    ”Det är ett enkelt tankeexperiment. Vad är din åsikt om det?”

    Jag är basalt empiriker och nöjer mig normalt mig med att försöka förstå världen som den är, varför jag avstår från att bilda mig någon åsikt rörande ditt tankeexperiment. Skälet till att jag i inlägget uppmärksammar det av SMHI-beskrivna tankeexperimentet rörande en atmosfärsfri jord är att IPCC åberopat det som förstahandsbevis för att det existerar växthuseffekter av högst betydande storlek.

  56. Gösta Pettersson

    #39 Leif Åsbrink

    Jag skrev för några timmar sen ett långt svar till din kommentar, men det förefaller inte ha blivit publicerat. Den här gången förmodligen för att jag själv lyckats radera i stället för att publicera svaret.

    Det gick ut på att du tar upp ett helt annat problem (växthuseffekters uppkomst) än det jag behandlar i mitt inlägg. I morgon (rättare sagt senare idag) ska jag försöka rekonstruera mitt svar, så giv dig till tåls. Den väntar aldrig för länge som väntar på godis. Du ska få ditt svar.

  57. Lars Cornell

    GöranA #51, Bjarne #49, GöranA #35
    Ni använder ordet ”markytan”. 90% av solens strålar gå ned i haven och värmer dess översta yta. Därför borde ni nog använda ordet ”havsytan”. Merparten av energin går uppåt i form av vattenånga som är ett annat sätt att lagra energi. När den återbildas till vattendroppar i molnen återfår man strålning. Men det kan ju vara rätt tänkt ändå.

  58. Lars-Eric Bjerke

    Molekylerna i en gas i en hög adiabatisk termos bör enligt mig få samma temperatur i hela termosen och inte samma energi, viket jag uppfattar att Gösta P. anser. Detta baserat på att värmetransporten alltid går från varmt till kallare. Har jag uppfattat Gösta P korrekt?

  59. Sten Kaijser

    Den här diskussionen är ju riktigt intressant. Om vi tänker oss en 100 meter hög termos, alltså ett rör med arean 1 kvadratmeter, fylld med luft.

    Vi vet att trycket avtar med höjden, men hur är det med temperaturen.

    Några hävdar bestämt att temperaturen blir densamma i hela röret, andra att vi får en gravitationell temperaturgadient.

    Johan M antyder att hans simulering inte hittar någon gradient.

    Jag ”lyssnar” på argumenten från båda sidor och tycker att de är bra. Jag avvaktar : )

  60. GoranA

    #56 Lars Cornell
    Om du läst min första kommentar #20 så inser du att jag är fullt medveten om din kritik.
    Jag skrev

    ”Om jag har förstått det rätt så beräknas den totala naturliga växthuseffekten till ungefär 33°C. Denna temperatur får fram genom att ta differensen mellan en beräknad temperatur från temperaturdata från termometrar och en beräknad temperatur från den medeleffekt som når Jorden.
    I det första fallet används ett snurrande jordklot vars yta till stor del består av hav is och land och en atmosfär som till en liten del består av IR-aktiva gaser. I det andra fallet räknar man på ett jordklot utan hav, is och atmosfär.
    Trots de uppräknade skillnaderna så antas hela temperaturskillnaden uppkomma p g a de IR-aktiva gaserna i atmosfären. Jag kan tycka att borde försöka beräkna den temperatur som uppstår i på Jorden i det fallet man bara ta bort de IR-aktiva gaserna från atmosfären jämfört med en vår naturliga Jord.”

    Jag anser att det inte går att jämföra en två så vitt skilda system som vår naturliga Jord med en utan atmosfär och påstå att skillnader endast beror växthusgaser vara eller inte.

    Jag anser dock att det räcker att införa en atmosfär utan växthusgaser så kommer den planeten att skilja sig i temperatur från en utan atmosfär oberoende av växthusgaser.

    Inför vi sedan att 70% av ytan består av hav så får det ytterligare konsekvenser för Jordens medeltemperatur oberoende om vattenångan räknas in som en växthusgas eller inte.

  61. GoranA

    #34 Leif Åsbrink
    Du skriver
    ”Därtill kommer att jorden förändras. En jord utan växthusgaser skulle inte kunna ha några moln eller någon växtlighet.”

    Kan du förklara närmare. Det är kanske självklart men jag ser inte det sambandet. Jag är medveten om vad som gör dessa molekyler till växthusgaser men om vi tänker oss att vi frikopplar den egenskapen från dessa molekyler men att de får behålla övriga egenskaper.
    Vad är det då som hindrar att moln kan bildas eller växtlighet att bildas?

  62. Lars-Eric Bjerke

    # Sten Kaijser,
    Johan M har ju med värmeledning d.v.s. molekylernas kollisioner i sin modell, men inte strålning, deras vibrationer. Gravitationen påverkar inte strålningen men något värmeledningen, molekylerna hastighet, vilket enbart medför att molekylerna högt i termosen får högre lägesenergi än de där nere, men samma temperatur. Värme går ju från varmt till kallare om inget arbete tillförs.
    Hoppas du får fler svar.

  63. Sten Kaijser

    Lars-Eric B #62

    tack! Jag tror att jag skulle föredra en ädelgas, exempelvis bara Argon, som nog är ganska nära en ideal gas, där det knappast finns någon annan energi ärn kinetisk.

    Man kunde också fundera på om det är någon skillnad om det är lågt tryck innen i röret, jämfört med större tryck.

    Som jag ser det så hos en molekyl som rör sig vertikalt sker det ett utbyte av kinetisk och potentiell energi. Det talar för en gravitationseffekt, d.v.s att det blir något varmare längst ner, men sedan sprider sig värmen med kollissioner uppåt, eller?

  64. Gösta Pettersson

    #39 Leif Åsbrink

    Negligera min kommentar i #56. Mitt svar på #39 blev tydligen publicerat (efter ovanligt lång tidsfördröjning med effektueringen av ett redigeringstillägg) pch återfinns som #54. Du har redan fått mitt svar.

  65. Lars-Eric Bjerke

    #63 Sten Kaijser,

    Vi tycks ha samma åsikt d.v.s. sluttillståndet i termosen blir en jämn temperaturfördelning med eller utan gravitation. Gravitationen påverkar däremot fördelningen av molekyler i höjdled och därmed trycket.
    Antalet molekyler borde påverka hur snabbt värmen sprider sig men inte sluttillståndet som en jämn temperaturfördelning.
    Hoppas att de verkligt kunniga bringar klarhet i varför/om vi tänker fel.

  66. Thomas P

    Det tycks i alla händelser som GP accepterar Manabes beräkningar. De kom fram till en klimatkänslighet på 2 grader, så i brist på mer direkta svar antar jag att GP också ligger ungefär där.

  67. Lars-Eric Bjerke

    #66 Thomas P

    Menar du ECS eller TCR? Fint om du vill svara på Sten Kaijsers fråga i #63 och #65?

  68. Gösta Pettersson

    #66
    ”Det tycks i alla händelser som GP accepterar Manabes beräkningar”

    Det ha jag aldrig sagt något om. Vad jag säger är att inte ens den Nobelprisade GHE-proponenten Wanabes modell ger något stöd för IPCC:s påstående att det finns en 32-gradig naturlig växthuseffekt. Modellen framställer tempeaturskillnaden mellan utstrålningsskiktets -18 °C och jordytan som en gravitationell effekt. Modellen godtar och bygger på att troposfärens temperaturer upp till cirka 8 km:s höjd (vid 300 ppm CO2) bestäms av en ”konvektiv jämvikt” i enlighet med Maxwells och Loschmidts idéer.

  69. Thomas P

    #67 Manabe beskriver vad vi vanliga dödliga kallar för växthuseffekten. De där 8 km du drar upp är just en effekt av växthusgaser, att atmosfären på lägre höjd är så ogenomskinlig för IR att istället konvektion tar över för värmetransport. När vi ökar mängden växthusgaser flyttar denna punkt gradvis uppåt och markytan blir varmare.

    Om du inte håller med Manabe och alla andra klimatforskare kanske du kan säga vad du anser jordens temperatur skulle vara
    1. Utan växthusgaser.
    2. Med dubblad CO2-halt.

    Det är sådant som är relevant, inte om du väljer att kalla växthuseffekten för en ”gravitationell effekt”.

  70. Sten Kaijser

    Hej igen Lars-Eric,

    jag är inte helt säker på vad jag ska tro. Jag tänker mig alltså mitt 100 meter höga rör, fyllt med argon. I botten på röret har jag ”i mitten” en elslinga som kan värma luften direkt ovanför.

    Jag kör igenom lite ström och får lite varmluft ”mitt i röret”. Den varma gasen stiger upp. Den utvidgar sig och avkyls. Men vad händer sen. Är den tillräckligt varm kanske den når upp till taket och lägger sig som ett varmt lock högst upp.

    Jag misstänker att man får köra som Johan M ”en simulering”.

    Och hur länge ska man vänta innan det stabiliserat sig?

  71. Lars-Eric Bjerke

    #58 Sten Kaijser

    I sin kommentar #50 till sitt första inlägg skriver Gösta P.
    ”Med statistisk-mekanisk metodik har man funnit att temperaturgradienter, snarlika den naturligt observerade, på grund av den gravito-termala effekten uppstår även i energetiskt slutna system (ingen uppvärmning underifrån, ingen energitransport), samt oberoende av om de tänkta atmosfärerna innehåller växthusgaser eller ej. Tydligare än så kan man knappast visa att temperaturgradienten representerar en gravitationseffekt.”

    Vad jag förstår strider detta mot termodynamikens andra grundlag, som för ovanstående adiabatiska system säger att ett sådant i termisk jämvikt är isotermt precis som Johan M har visat. Jag anser att i detta fall är inte Gösta P´s påstående ovan korrekt.

    I det verkliga fallet här på jorden har ju gravitationen betydelse för hur tjockt vårt luftskikt är. Det är föstås trivialt att denna gravitationseffekt påverkar lapse rate.
    Jag tycker det räcker att visa att Gösta P´s teori inte gäller i det adiabatiska fallet för att teorin ska falla.

  72. Lars-Eric Bjerke

    …. forts
    Rättelse: jag menade din kommentar #68 inte 58.

  73. bength

    Temperaturskillnaden mellan ”utstrålningsskiktet” och jordytan beror av både gravitationen och atmosfärens IR-absorberande egenskaper. Man skulle matematiskt kunna skriva det som: Tdiff = dT/dH * H, där dT/dH är temperaturgradienten som beror av g, och H är utstrålningsskiktets höjd som beror av mängden växthusgaser i atmosfären (inkl. vattendroppar i moln). Sätts någon av dessa till noll så blir temperaturskillnaden, Tdiff, också noll.

    Om man vill kalla temperaturskillnaden för en gravitationseffekt i stället för en växthuseffekt, så kan man ju göra det. (Även en annan naturkonstant, luftens cp, spelar roll). Formeln ovan som visar ett lika stort beroende av båda gäller hur som helst. Men att från detta dra slutsatsen ”att den naturliga växthuseffekten är av insignifikant storlek” saknar helt grund.

    Detta var ett försök att sammanfatta frågan som jag uppfattat den. Rätta mig gärna om jag missuppfattat något. Jag har använt mig av de starka förenklingar som redan var gjorda i huvudinlägget.

  74. #54 Gösta Pettersson

    Hej Gösta!

    Det verkar som ett missförstånd. När jag läser denna mening: ”Såväl teoretiska analyser som empiriska observationer ger belägg för att det existerar en gravitationsbetingad 32-gradig atmosfärseffekt som återspeglar luftmolekylernas massa och värmekapacitet, utan någon som helst påvisad kvantitativt signifikant relation till molekylernas spektrala egenskaper och förmåga att absorbera värmestrålning.” tolkar jag den som att den 32-gradiga atmosfärseffekten inte signifikant beror av atmosfärens spektrala egenskaper.

    Detta tolkar jag som att marktemperaturen inte skulle påverkas signifikant av t.ex. en fördubbling av CO2-halten i atmosfären.

    Nu skriver du ”För avslutningsvis kan jag citera dig i ett avseende där vi (och förhoppningsvis alla) är helt överens: ’JA! CO2 påverkar energibalansen (dvs temperaturen)’ ”

    Det gör mig förbryllad – du menar antingen att jag misstolkat uttrycket ”gravitationsbetingad 32-gradig atmosfärseffekt” eller ”utan någon som helst påvisad kvantitativt signifikant relation till molekylernas spektrala egenskaper” För mig låter båda sakerna mycket väldefinierade och din slutsats ovan oförenlig med att ”CO2 påverkar temperaturen vid marken.” Kan det vara så att du menar att inverkan av CO2-halten är så obetydlig att den inte är signifikant ? Hela klimathysterin handlar ju om att understiga 1,5 grader vilket vi alla är väl medvetna om så ”inte signifikant” tolkar jag som liten i jämförelse med 1,5 grader.

    Att det finns åtskilliga skeptiker som publicerat artiklar där dom visar att CO2 inte påverkar temperaturen alls är ett beklagligt faktum. När jag klickar på länken THE HOCKEY SCHTICK finner jag denna text: ”We will derive the entire 33°C greenhouse effect using the 1st law of thermodynamics and ideal gas law without use of radiative forcing from greenhouse gases, nor the concentrations of greenhouse gases, nor the emission/absorption spectra of greenhouse gases at any point in this derivation”
    Du skriver: ”Några sådana skeptiker känner jag inte till. Ovanstående tjugotal forskare har grundat sina slutsatser på erkänt seriösa fysikaliska teorier och samband, använda inom meteorologin för deras goda beskrivning av empiriska observationer. Och mig veterligt har ingen av dom hävdat att dom ’bevisat’ att CO2 inte kan påverka strålningsbalansen.” Jag tycker att ”nor the concentrations of greenhouse gases” är ekvivalent med att säga att CO2 inte alls påverkar temperaturen.

  75. #61 GoranA

    En atmosfär utan växthusgaser är fysikaliskt tänkbar – låt vara fullständigt orealistisk. Skulle kunna vara kväve och lite ädelgaser.

    Moln kräver molekyler som kan binda till varandra med svaga bindningar och det kräver molekyler med minst tre atomer och sådana är aktiva i infrarött.

    Vill man skapa en rimlig bild av vad referensobjektet är så är det bättre att säga att man jämför med ett klot helt utan atmosfär, säg 1 m i diameter av ett material med oändlig ledningsförmåga och målat med en färg som ger samma albedo som jorden i synligt såväl som infrarätt. Jorden utan växthusgaser är en orimlighet.

    Klotet kunde vara ihåligt och av koppar och innehålla lite absolut rent vatten helt utan andra gaser. Det skulle ha ett finmaskigt nät på insidan så att det fungerade som en ”heat pipe” och därmed fick i det närmaste oändlig värmeledningsförmåga.

    Klotets storlek är inte relevant för värmebalansen och huruvida det roterar eller ej spelar ingen roll.

  76. Lars Cornell

    #65 Bjerke.
    ”Antalet molekyler borde påverka hur snabbt värmen sprider sig …”
    Där delar jag inte din uppfattning. Varför skulle det vara så. Däremot har det betydelse hur snabbt de ’flyger’ omkring. Se föregående artikel ”den-gravito-termala-effekten” #5, #36 och #43. Se även,
    https://sv.wikipedia.org/wiki/Ljudhastighet

  77. Gösta Pettersson

    #74 Leif Åsbrink

    ”Att det finns åtskilliga skeptiker som publicerat artiklar där dom visar att CO2 inte påverkar temperaturen alls är ett beklagligt faktum. ”

    Att skeptiker lagt fram modeller som visar att man tillfredsställande kan beskriva empiriska resultat (t ex troposfärens temperaturgradient) utan ta hänsyn till växthuseffekter är för mig inte samma sak som att man visat eller påstått att det inte finns några växthuseffekter. Som statistiskt skolad empiriker inser jag att den enda slutsats man kan dra är att växthuseffekterna befunnnits vara av insignifikant storlek. Det inser och påpekar även Ingemar Nordin i sin kommentar #25.
    Det är möjligt att enstaka skeptiker i sådana sammanhang faktiskt dragit slutsatsen att växthusgaser har noll effekt. Men jag har inte sett någon göra det. Och skulle jag göra det, så skulle jag be Gud förlåta dem sådana rader. Men jag skulle inte oroas ett skvatt för att deras dåliga formuleringar skulle kunna dra löje över klimatskepticismen.

    ”Kan det vara så att du menar att inverkan av CO2-halten är så obetydlig att den inte är signifikant ? ”

    Ja, det är precis vad jag lägger in i formuleringar typ att den 32-gradiga ”naturliga växthuseffekt” som IPCC anser föreligga i själva verket tankeexperimentellt har befunnits vara av insignifikant storlek.

    ”Detta tolkar jag som att marktemperaturen inte skulle påverkas signifikant av t.ex. en fördubbling av CO2-halten i atmosfären.”

    Det är en fullt rimlig tolkning av de argument och slutsatser som redovisas i inlägget , närliggande vad jag säger i sista stycket av min kommentar #32.

  78. Gösta Pettersson

    #73 bength
    ”Temperaturskillnaden mellan ”utstrålningsskiktet” och jordytan beror av både gravitationen och atmosfärens IR-absorberande egenskaper.”

    Jag känner till hypotesen att växthusgaser förhöjer ”utstrålningsskiktets” läge, men har inte träffat på några empiriska eller modellmässiga belägg för att så är fallet.

    Manabes av nutida klimatologer favoriserade modell föreskriver att växthuseffekten uppstår på grund av att den troposfäriska temperaturgradienten uppvisar radiativt betingade avvikelser från konstans på höjder över 8 km. Den hänför alltså inte växthuseffekten till någon förhöjning av ”utstrålningsskiktet” med dess -18 °C, utan till skeenden i regioner långt ovanför ”utstrålningsskiktet” (dvs. långt ovanför de regioner där den huvudsakliga utstrålningen sker). Vilket betyder att utstrålningsskiktet höjd outsagt förutsätts vara konstant och opåverkad av växthusgaser.

    Även GHE-opponenter anser utstrålningsskiktets läge vara oberoende av atmosfärens IR-absorberande egenskaper och kalkylerbart enligt statistisk-mekaniska principer som enbart tar hänsyn till massa och energi.

    Såväl GHE-opponenter som GHE-proponenter förefaller därför i nuläget vara eniga om att det saknas fog för ditt förmodande att temperaturskillnaden mellan ”utstrålningsskiktet” och jordytan inkluderar signifikanta bidrag från troposfärens IR-absorberande egenskaper.

    Det är möjligt att man i framtiden genom satellitbaserade mätningar kan befästa att sådana bidrag föreligger, t. ex. på grund av en växthusgasframkallad förhöjning av utstrålningsskiktets belägenhet. Men det förefaller mig föga troligt att bidragen kommer att vara av sådan storlek att de ger stöd för IPCC:s påstående att det jorden uppvisar en 32-gradig naturlig växthuseffekt.

    Mitt grundtips är att växthuseffektsbidrag till den 32-gradiga atmosfärseffekten även framledes kommer att befinnas vara av insignifikant storlek. Gardera med att de kanske kan befinnas vara i storleksorningen 2 °C. Och tack för din analytiska klarsynthet som tvang mig att tänka till ett varv extra.

  79. GoranA

    #34 Leif Åsbrink
    Du skriver
    ”Därtill kommer att jorden förändras. En jord utan växthusgaser skulle inte kunna ha några moln eller någon växtlighet.”

    #61 GoranA
    Jag ber dig förklara närmare. Vad moln och liv har med egenskapen växthusgas att gör med moln och liv.

    #75 Leif Åsbrink
    Du skriver
    Moln kräver molekyler som kan binda till varandra med svaga bindningar och det kräver molekyler med minst tre atomer och sådana är aktiva i infrarött.

    Här kan jag invända att HF (väteflurid) har relativt starka bindningar men är en tvåatomär gas och bör vara då inte vara en växthusgas.

    I en annan värld skulle haven och atmosfären kunna bestå av vätefluorid. Antag en planet med en temperatur där HF inte stelnar. Vi skulle då ha en planet med två energilagrande massor och du påstår att på en sådan planet skulle markytan ha samma temperatur som en planet utan hav (HF) och atmosfär (HF)
    .
    Sedan skapar du en ny modell som jag uppfattar som en kropp utan möjlighet till lagring av energi vilket av en kropp är i motsats till den planet som jag försöker argumentera för.

    Jag undrar fortfarande vad kopplingen är mellan liv och växthusgaser.

  80. Bjarne Bisballe

    #75 Leif Åsbrink
    Det rette referenceobjekt må være en ’tænkt’ roterende klode med atmosfære, men uden drivhusgasser, dvs en atmosfære med kun med de ’almindelige’ gasser N2 og O2, for forskellen er alene ’drivhusgasser eller ej’ .

    GoranA hævder at energi ophober sig fra dag til dag på en sådan klode i #51 (”Det tar längre tid kyla atmosfären under natten än det tar att värma den under dagen”), mens William Happer siger i #22 ”The atmosphere cannot store heat from one day to the next. If it could, the surface would be warmer and warmer an warmer and warmer….. ” Men hvem har ret? Det er vigtigt at vide, for hvis Happer har ret, er den globale opvarmning 33K, (255K til 288K) men hvis GoranA har ret er den globale omvarmning mindre end 33K og spørgsmålet bliver så: Hvor meget mindre?

  81. Sten Kaijser

    Vi vet att den huvudsakliga värmetransporten upp till ”utstrålningshöjden” sker med latent värme, d.v.s. med vatten som avdunstat, främst från tropiska hav.

    Det jag inte vet eller förstår är varför ”en så stor andel” av vattenångan kondenserar på just ”den höjden”.

    Vilka egenskaper hos vattnet och/eller atmosfären får vattnet att kondensera på lämplig höjd.

    Oavsett vad temperaturskillnaden beror på så är det ju bra att temperaturen är lagom på lämplig höjd för att ge lämplig temperatur vid ytan.

  82. Lars-Eric Bjerke

    #76 Lars Cornell,
    ””Antalet molekyler borde påverka hur snabbt värmen sprider sig …”
    Där delar jag inte din uppfattning.”

    Jag skrev om värmetransport i en adiabatisk termos. Den består av värmeledning och värmestrålning. Vad gäller värmeledning är denna beroende av antal molekyler i termosen och hur ofta de kolliderar (hastigheten). Jämför med att man pumpar vacuum för att förbättra isolerförmågan.

  83. Thomas P

    #78 ”Även GHE-opponenter anser utstrålningsskiktets läge vara oberoende av atmosfärens IR-absorberande egenskaper och kalkylerbart enligt statistisk-mekaniska principer som enbart tar hänsyn till massa och energi.”

    Så energin skall stråla ut från 8000 meter höjd, även om atmosfären inte har några gaser i sig som kan stråla ut IR. Samtidigt skall utstrålningen från marken inte stråla rätt ut i rymden trots att det i denna tänkta atmosfär utan växthusgaser inte finns något som absorberar IR. Det låter ju rimligt…

    Dessutom har man konstaterat att tropopausen flyttar sig uppåt med ca 5 m/år. Det är ett empiriskt faktum.
    https://www.science.org/doi/10.1126/sciadv.abi8065

  84. Lars Cornell

    Bjerke #82
    Nu blandar du samman två begrepp, mängd och hastighet.
    En häst kan förflytta att lass med en viss hastighet.
    Hundra hästar kan förflytta hundra lass, men det går inte fortare.
    Hundra hästar kan fylla en lada med hö fortare än vad en häst kan.

    Hur fort en häst springer beror till viss del på temperaturen men inte hur många de är.

  85. Lars Cornell

    #81 Kaijser
    Det tycker jag var intressant. Men är frågan rätt ställd?
    Vattenångan från haven kondenserar på alla höjder i troposfären är min uppfattning.

    Dessutom har vi, tror jag, ett accelererande förlopp. När vattenångan kondenserar börjar utstrålningen (som från nästan svart kropp) som kyler. Det får kondensationen att accelerera. När det sker, men det tar litet tid (tid * hastighet = vertikal sträcka), bildas de stora molnmängderna.

    När man kommer upp till några tusen meters höjd är vattenhalten i ångan så låg att det inte blir så mycket mer moln av det.

  86. #79 GoranA

    Nåväl, HF, vätefluorid har ett starkt dipolmoment och därför några starka absorptionslinjer i rotationsspektrum. De är smala och vid låg energi, snarare mikrovåg än infrarött så de betyder inte mycket. Där finns också vibrationslinjer men dom ligger på för hög energi för att ha någon betydelse.

    HF är emellertid känt för att vara en icke ideal gas. På grund av sitt starka dipolmoment bildar den gärna dimerer, trimerer, tetramerer,… Dessa polyatomära molekyler är starkt aktiva i infrarött. T.ex. denna: https://www.synchrotron-soleil.fr/en/news/cyclic-ground-state-structure-hf-trimer-revealed-jet-ailes-device En HF planet skulle definitivt ha växthuseffekt såvida inte atmosfärstrycket vore extremt lågt – men då skulle nog inte vätskefasen kunna finnas.

    Möjligen skulle en atmosfär av huvudsakligen helium och lite HF kunna ge flytande HF och endast monomeren av HF i gasfasen. Växthuseffekten skulle i så fall bli extremt svag.

    Du föreslår att man borde försöka beräkna den temperatur som uppstår på Jorden i det fallet man bara tar bort de IR-aktiva gaserna från atmosfären jämfört med en vår naturliga Jord. Det skulle vara ett mycket besvärligt och godtyckligt projekt. Skall molnen vara kvar så att albedo blir oförändrat? Skall vattenångan fördelas på samma sätt som på den verkliga jorden så att energitransporten med latent värme skall fortgå? Det skulle inte alls stämma med den lägre temperatur som skulle uppstå.

    Hur man än gör blir det alldeles galet på något sätt. Den ”vanliga” modellen, en planet med jordens albedo, 0,3 och helt utan atmosfär innehåller också förutsättningen att temperaturen skall vara lika över hela ytan året runt. Det innebär att värmeledningsförmågan hos den tänkta växthusgasfria planeten är oändlig och att värmekapaciteten är mycket stor så att årstidsvariationerna är helt utjämnade.

    En verklig stenplanet utan atmosfär och med jordens geometri och rörelsemönster skulle få en extremt hög temperatur sommartid vid polerna när solen står ungefär 30 grader över horisonten dygnet runt. Vintertid skulle temperaturen bli extremt låg efter ett halvår helt utan sol.

    Hur man än gör blir resultatet missvisande. Det är därför jag föreslår en referenskropp som tydligt visar vad man menar. Det är alltså temperaturen hos en kropp helt utan atmosfär med samma energibalans som jorden. Ett teoretiskt objekt, men fullt realiserbart. Någon energilagring blir det inte. Man får räkna ut årsmetelvärdet av energiflödet och och räkna ut en temperatur. Medeltemperatur blir det inte, men det man vill jämföra med är årsmedelvärdet av den strålning jorden tar emot.

    Jag tycker min diskussion med Gösta tydliggör varför jag vill avföra begreppet ”den 33-gradiga växthuseffekten.” Siffervärdet dyker upp här och där i galna teorier, t.ex. hos Lord Moncton. Genom att presentera ett realiserbart fysiskt obnjekt tydliggör man enligt min mening att det är strålningsbalansens årsmedelvärde som avses. Inte den ”verkliga” temperatur jorden skulle få ”utan växthusgaser.”

    Din fråga ”Jag undrar fortfarande vad kopplingen är mellan liv och växthusgaser.” besvarar jag med en motfråga: Kan du tänka dig någon form av liv utan vatten?

    Jag vet att det finns science fiction som skissar på högtemperaturliv baserat på kisel i stället för kol men jag har svårt att tro på det – och utan atmosfär och hav av något slag blir det alldeles för mycket fiction i min smak. Jag är också rätt övertygad om att de vätskor och gaser man skulle kunna tänkas hitta på kommer att visa sig vara växthusgaser. Liv kräver komplex kemi. Då måste komplexa molekyler kunna röra sig i vätska eller gas.

  87. Adepten

    #86 Leif Åsbrink

    Göran Åkesson har med sin härledning visat att Solinstrålningen räcker till för att värma upp jordklotet. Växthusgasmodellen faller därmed bort som förklaring till jordens klimat.
    http://www.diva-portal.se/smash/get/diva2:421615/FULLTEXT06.pdf
    Görans teori är intressant om den kan ledas i bevis? Kan du se om det är något fel i hans bevisföring?

  88. Adepten

    #86 Leif Åsbrink

    Länken jag sände iväg blev aningen fel 🙁 Här är rätt länk:
    https://klimatsans.com/2021/06/24/solen-utan-vaxthuseffekt/

  89. GoranA

    #86 Leif Åsbrink

    Jag är tacksam att du tar dig tid att försöka besvara mina frågor.

    Som svar på din motfråga så kan jag inte tänka mig ett liv utan vatten

    Jag anser dock inte att du tydligt förklarar vad den specifika egenskapen” växthusgas” har med moln och liv att göra. Det finns ju växthusgaser som freoner som är helt skilda från liv moln, antar jag.

    Att vatten och koldioxid är essentiellt för högre liv är sjävklart. Koldioxid som en friflytande kolkälla i atmosfären och hydrosfären, vatten som bl a ett suveränt transport- och lösningsmedel. Jag ser dock inte kopplingen till deras egenskaper som växthusgaser.

    För återgå till grundfrågan som gällde om en planet utan atmosfär (A-)och en planet med atmosfär (A+) men utan växthusgaser har samma medeltemperatur. I fallet med Jorden som exempel skulle båda planeterna ha temperaturen -18°C.

    För mig är det troligt att en A+ får en högre medeltemp än en A-.

    A- fungerar mer eller mindre som en svartkropp och strålar ut lika mycket energi som den mottager. Den värms snabbt och svalnar lika snabbt under solens uppgång och nedgång. När solen gått ner har planetytan förlorat i princip all energi som den mottog under dagen.

    A+ kommer att föra över en del av sin energi under dagen till atmosfären via ledning, atmosfären värms olika p g a ojämn belysning, solhöjdens variation. Uppvärmda gaspaket stiger i luften som ger upphov till vindar längs marken som ökar på överföring och inblandning av energi från planetytan till atmosfären. Planetytan för A+ blir inte lika varm under dagen som A-. Vid solnedgången kommer det finnas energi kvar på A+. Under natten kyls atmosfären via ledning mot den kallare planetytan.

    Här antar jag att uppvärmningen av atmosfären på A+ är effektivare via förstärkt ledning (vindar) och konvektion (blandning) under dagen jämfört med avsvalningen under natten med svagare vindar. Är avkylningen tillräckligt ineffektiv skulle det innebära att morgontemperaturen är högre för A+ än A-.

    Sammantaget skulle A+ värmas jämfört med A- och möjligen alltmer tills den allt varmare planetytan motverkas av en den ökade temperaturen genom en allt effektivare utstrålning, F =sigma*Temp^4

    I ett nästa skulle man kunna lägga till 70% havsyta till A+ vilket skulle förbättra energilagringen och ytterligare utjämning av temperatur och en sjunkande utstrålning med en ökad medeltemperatur som följd.

  90. Björn

    GoranA [89]; Ett enkelt svar på din framställan, är att utan vattenånga som populärt kallas för växthusgas, bildas inga moln. Den oundvikliga konsekvensen med vatten är alltså vattenånga och därmed enligt fysikens lagar, kondensering till moln.

  91. #88 Adepten

    Matematik är besvärligt. Dubbelintegraler till exempel. Funktionen när man lämnar ekvatorn är inte en cosinus, man måste räkna ut vinkeln till solen från varje punkt.

    Fysik är mycket enklare. Tänk dig att man mäter strålningen bakom jorden. Hur stor diameter har skuggan som jorden ger? All strålning som fattas i skuggan av jorden måste ha absorberats av jorden.

    Halva ytan absorberar således 1370 W/m^2

    Göran Åkesson har gjort ett elementärt räknefel.

  92. GoranA

    90 Björn
    Jo jag förstår att utan vattenånga kondenserar till vatten på kondenskärnor som till vattendroppar som bildar moln.

    Detta kan kopplas ihop med att vatten är ett ämne med en unikt hög sm p och k p, trots sin litenhet, vilket gör att vatten naturligt kondenserar och förångas i en samverkan mellan rådande temperatur och det relativ mättnadstrycket.

    På Jorden råder sådan förhållanden att temperaturen sjunker på stigande höjder vilket gör vattnets mättnads tryck minskar med stigande höjd vilket möjliggör kondensation om där finns kondensationskärnor att tillgå.

    Vattnets höga sm p och k p beror på att vattenmolekyler kan binda till varandra m h a vätebindningar som uppstår då en starkt elektronegativ atom som syre har en kovalent bindning till väte. Via ett fritt elektronpar på syret i vattenmolekylen kan elektronparet ge upphov till en vätebindning till ett väte i en annan vattenmolekyl. Vätebindningen är den starkaste intermolekylära bindningen åtminstone när det gäller små molekyler. Van der Waals bindningen växer med kontaktyta och molmassa och kan bli mycket stark i jättemolekyler som skikten i grafit (grafen).

    Endast vätefluorid kan skapa en starkare vätebindning, men i genomsnitt endast två per molekyl. Vatten som är mer symmetrisk kan ingå i fyra stycken vätebindningar.

    Vattenmolekylen har två fria e-par och bundna väteatomer. Vatten får därmed högre sm p och k p än vätefluorid trots att varje vätebindning är svagare.

    Vätebindning ger också upphov till ytspänningen, vattnets höga värmekapacitet. vattnets höga smält- och förångningsentalpier. Vattenmolekylen är också en dipol vilket gör att den kan samverka med joner och lösa vissa salter salter på ett bra sätt dock inte alla.

    Ingen av dessa egenskaper kopplar jag ihop med att vatten molekyl är en växthusgas.

    Definitionen för en växthusgas är väl att den kan absorbera och emittera infraröd strålning.

  93. #89 GoranA

    ”Jag anser dock inte att du tydligt förklarar vad den specifika egenskapen ’växthusgas’ har med moln och liv att göra. Det finns ju växthusgaser som freoner som är helt skilda från liv moln, antar jag.”

    Att B följer av A betyder inte att A följer av B. Alla gaser som består av mer än två atomer är växthusgaser därför att sådana molekyler inte bara kan vibrera genom att bindningens längd varierar. Dom kan också vibrera genom att vinkeln mellan bindningar varierar. Böjsvängningar har lägre energi än sträcksvängningar och hamnar (nästan alltid, eller möjligen alltid) i det område där strålningen från jordiska temperaturer är stark. Det finns tvåatomiga polära molekyler som är växthusgaser. När bindningen är tillräckligt svag och atomerna tillräckligt tunga får sträcksvängningen tillräckligt låg energi. Är atomerna dessutom olika får molekylen dipolmoment och kan växelverka med det elektromagnetiska fältet (absorbera). Jodmonoklorid är ett exempel. Den absorberar vid cirka 395 cm-1.

    ”För återgå till grundfrågan som gällde om en planet utan atmosfär (A-)och en planet med atmosfär (A+) men utan växthusgaser har samma medeltemperatur. I fallet med Jorden som exempel skulle båda planeterna ha temperaturen -18°C.”

    Nej, ingen av dom skulle ha medeltemperaturen -18°C. (A-) skulle ha extremt olika temperatur på polerna sommar och vinter. Ju mer temperaturerna skiljer desto mer energi strålar ut totalt. (A+) skulle rimligen få cirkulation i atmosfären på grund av skillnader i marktemperatur och då skulle temperaturfördelningen bli jämnare, utstrålningen lägre och temperaturen högre än i fallet (A-).

    Endast en låtsasplanet med samma temperatur över hela ytan hela dygnet, året runt, skulle ha den berömda temperaturen -18°C – och endast om dess albedo i infrarött vore 0,000 – dvs en ideal svartkropp. Emissiviteten beror av våglängd, vinkel och polarisation och är inte alls 1,0 för verkliga matrial. Jag har inte data tillgänliga, men albedo 0,95 i infrarött är nog mer rimligt än 0,000. Effekten W/m2 är given och således måste temperaturen öka med fjärde roten ur 1/0,95 =1,0129 så i stället för 255K får man 258K eller -15°C grader ungefär.

    Den 33-gradiga växthuseffekten är bara till för att säga att det är en avsevärd atmosfärisk effekt som inte skulle finnas ifall inga växthusgaser fanns i atmosfären så att allmänheten får ett ungefärligt hum om att atmosfären har betydelse för temperaturen. Att det blir just 32 grader (eller 33) vid nuvarande sammansättning av atmosfären beror givetvis på den gravito-termala effekten. Skulle vi tillföra mer massa till atmosfären utan att ändra dess sammansättning skulle temperaturen öka, det strålande skiktet är (ungefär) medelvärdet av den höjd där en infallande foton från världsrymden har haft 50% chans att bli absorberad (medelvärde över spektrum, infallsvinklar och tid). Med mer atmosfär ökar den höjden.

    Det enda som har vetenskapligt intresse är hur den totala atmosfärseffekten ändras när de spektrala egenskaperna ändras lite genom förändring av de välblandade växthusgaserna. Dvs alla utom vatten (såvitt jag vet.)

  94. Bjarne Bisballe

    #93 Leif Åsbrink
    Tak for en god forklaring

  95. GoranA

    #93 Leif Åsbrink

    Jag får ge mig gällande kopplingen mellan vatten som växthusgas och molnbildning. Du anser att du förklarar, men jag ser som sagt inte sambandet. Som jag ser det förklarar du utmärkt varför en molekyl blir en växthusgas men inte hur det främjar molnbildning eller för den delen varför dessa egenskaper är viktiga för livet.

    Om du menar att Jorden utan växthusgaser hade varit en isboll utan varken liv eller moln så kan jag förstå det.

    I min kommentar ovan #92 redogör jag för min förståelse när det gäller molnbildning där vattnets förmåga att bilda vätebindningar är centrala.

    Angående grundfrågan
    Om jag tolkar dig rätt så ger du mig rätt att A+ och A- inte är likvärdiga.

    Du ger mig rätt att ledning kan överföra energi från en solbelyst markyta till atmosfären.

    Du ger mig rätt att en A+; kan lagra energi i atmosfären, få en högre medeltemperatur, få en jämnare temperatur.
    Så om någon hävdar A+ och A- är likvärdiga så kan jag hänvisa till dig om att de har fel.

    Jag tror att ledning är en underskattad när det överföring av energi till eller ifrån atmosfären. Ledning sker i kontakt mellan materia och gaser anses vara dåliga ledare och används som isolering. Via vind och en enorm förstoring av markytan via växter (träd) bör den ändå få en betydande effekt.

    Det återstår alltså att bestämma växthuseffekten storlek om det nu är möjligt.

  96. Bjarne Bisballe

    #93 og #95
    Växthuseffekten storlek?

    Man kunne jo følge opskriften her og gå ud fra 258K i stedet for 255K.
    Hvad er din mening om denne artikel (12 sider), Leif Åsbrink?

    https://article.sciencepublishinggroup.com/pdf/10.11648.j.ijaos.20210502.12.pdf

  97. #77 Gösta Pettersson

    Hej Gösta!

    Nu tror jag att jag har helt klart för mig hur du tänker.

    Som Ingemar Nordin skriver i #25 är ju denna debatt lite vid sidan om huvudspåret i ”klimatdebatten” idag, men jag befarar att den kommer att bli aktuell i den kommande valrörelsen. Därav mitt engagemang.

    Henrik Svensson, vd för det oljeeldade reservkraftverket i Karlshamn skriver larmartikel i Dagens Industri.
    https://www.di.se/nyheter/larmet-valdigt-allvarligt-lage-for-elforsorjningen/

    Det är totalt oansvarigt enligt min mening att låta CO2-skräcken förhindra att man i tid ser till att Sverige kan producera tillräckligt mycket El även ifall det blir en mycket kall vindstilla vecka kommande vinter. Vindstilla i våra grannländer också. Eventuellt stopp på gas från Ryssland. SVK har roterande bortkoppling som sista utväg för att se till att elnätet inte kraschar. Skulle dom misslyckas och vi får en blackout kan det ta en vecka att få strömmen tillbaka. Vad som då skulle hända tror inte jag någon planerat för.

    Moderna hus med lätta väggar och god isolering kallnar fort ifall värmekällan stängs av. Är värmedistributionen till olika rum vattenbaserad måste allt vatten tappas ur elementen rätt snart. Även dricksvatten, diverse vattenlås och annat måste tömmas. Det fattar man inte omedelbart eftersom strömavbrott inte är någon extrem händelse normalt sett. Jag tror moderna fastighetsförvaltare inte längre har en portvakt/fastighetsskötare i varje fastighet. Personal från något servicebolag får åka runt och fixa. Hur gör dom när det inte går att tanka bilen längre? Och telefonnätet havererat? Det skulle kunna gå väldigt illa. Att se till att risken för blackout minimeras ser jag som mycket viktigt.

    I dag premierar kraftvärmebolagen värmeproduktion och elen kommer i andra hand. Detta kan ändras och man kommer troligen att behöva tillskott av fossil energi i kraftvärmeverken. Då behöver diverse lagar ändras. Så fort detta börjar diskuteras, som jag hoppas kommer att ske i valrörelsen, kommer de mest militanta alarmisterna att försöka stoppa ”nya fossila utsläpp.” Det kan bli en infekterad debatt – och i den kommer personer som läst din slutsats ”att det existerar en gravitationsbetingad 32-gradig atmosfärseffekt som återspeglar luftmolekylernas massa och värmekapacitet, utan någon som helst påvisad kvantitativt signifikant relation till molekylernas spektrala egenskaper och förmåga att absorbera värmestrålning.” att sprida den falska informationen att det är bevisat att CO2 inte har någon inverkan på jordens temperatur. Sådant sprids ju på nätet och ibland även på insändarsidor. Det eldar på alarmisterna som får skäl att misstro alla som är emot deras agenda.

    Vad du bevisar är att termodynamiken inte motsäger att CO2 kan sakna inverkan på temperaturen vid markytan. Mer konventionellt uttryckt. Man kan inte med termodynamiska argument motbevisa att det kan finnas återkopplingsmekanismer som (nästan) helt motverkar den uppvärmningseffekt som en CO2-ökning har i en i övrigt oförändrad atmosfär.

    Detta ditt generella påstående ”utan någon som helst påvisad kvantitativt signifikant relation till molekylernas spektrala egenskaper och förmåga att absorbera värmestrålning.” är falskt enligt min mening. Jag antar att du menar att nyckelordet är ”påvisad” – men då tycker jag du lurar dina läsare.

    Visst kan man ifrågasätta IPCC:s ”bevis” för att det är växthusgaserna som förorsakat den observerade temperaturstegringen. Något strikt bevis föreligger inte. IPCC anför att modeller som inte tar hänsyn till den antropogena CO2-ökningen inte kan fås att stämma med observationer. Det är inget bevis. Jag är övertygad om att det går att konstruera modeller där hela uppvärmningen har någon annan orsak. Solens väg genom rymden har ändrat den kosmiska omgivningen eller vad som helst som varken kan bevisas eller motbevisas.

    Däremot finns spektra av strålningen från jorden sedd från rymden. Vad som skulle hända om CO2 helt försvann är enligt min mening påvisat trots att experimentet inte kan utföras praktiskt. Den samlade kunskapen om strålningsfysik ger oss en säker bild av vad som skulle ske om CO2 momentant försvann ur atmosfären. Om detta säger du: ”Jag är basalt empiriker och nöjer mig normalt mig med att försöka förstå världen som den är, varför jag avstår från att bilda mig någon åsikt rörande ditt tankeexperiment.” Må så vara, men då borde du i alla fall ändra på din slutsats: ”Såväl teoretiska analyser som empiriska observationer ger belägg för att det existerar en gravitationsbetingad 32-gradig atmosfärseffekt som återspeglar luftmolekylernas massa och värmekapacitet, förutsatt att utstrålningen från jorden inte förändras. Alternativt förutsatt att höjden för det generaliserade utstrålningsskikt som motsvarar utstrålningen från jorden inte förändras.” Detta är ju den förutsättning som ditt resonemang bygger på och den är viktig!

    Jag kan inte tro att du är helt omedveten om hur spektrum från jorden ser ut när man tittar lodrätt från satellit eller flygplan. Jag kan heller inte föreställa mig att du är okunnig om vad som skulle hända om det inte funnits någon koldioxid i atmosfären enligt mycket väletablerad och 100% trovärdig forskning. Självklart vill jag inte påstå att man vet exakt vad som händer – det varierar i tid och rum och blir olika beroende på om man tittas på spektrum i någon vinkel från lodlinjen. Jag efterfrågar inte några detaljer. Jag vill bara att du skall medge att effekten skulle bli mycket kraftig och att du bör ändra påståendet om det du bevisar till att vara villkorat av att höjden för utstrålningsskiktet är oförändrad eller liknande.

  98. 96 Bjarne Bisballe

    Artikeln David Coe et al, 2021 bygger på felaktig analys. Se här:
    https://klimatupplysningen.se/klimatpandemier/#comment-419822

  99. #95 Goran A

    ”Det återstår alltså att bestämma växthuseffekten storlek om det nu är möjligt.”

    Det anser jag är fel. Växthuseffekten beror av godtyckliga antaganden. Det konventionella ligger nära de tankegångar som ligger till grund för resonemang om jordens strålningsbalans.

    Jag presenterade tanken på en realiserbar artificiell kropp som kunde tjäna som referens för att tona ner betydelsen av att man valt en ofysikalisk modell av jorden.

    Den konmventionella modellen med samma temperatur över hela ytan är vald för att den är begreppsmässigt enkel – inte för att den på något sätt är fysikaliskt rimlig.

  100. Adepten

    #91 Leif Åsbrink

    Du leder i bevis att Görans teori är falsifierad genom att utgår från att Jorden är en platt skiva, det imponerar inte på mig. Du lurar bara dig själv om du inte kan använda dig av en dubbelintegral för att bevisa att du har rätt.
    Förövrigt är vi överens, tror jag, om att ”klimatnödläget” inte är en bra arbetshypotes för att lösa mänsklighetens energiproblem.

  101. Gösta Pettersson

    #97 Leif Åsbrink
    ”Detta ditt generella påstående ”utan någon som helst påvisad kvantitativt signifikant relation till molekylernas spektrala egenskaper och förmåga att absorbera värmestrålning.” är falskt enligt min mening. Jag antar att du menar att nyckelordet är ”påvisad” – men då tycker jag du lurar dina läsare.”

    Int’ fan försöker jag lura någon, det vore fjärran från mitt kynne och mitt intresse att förstå verkligheten. Uttalandet gällde den 32-gradiga atmosfärseffekten och uttrycker min bedömning av det föreliggande kunskapsläget. Nyckelorden är ”kvantitativt signifikant”, baserat på de i inlägget redovisade argumenten för att den 32-gradiga temperaturskillnaden tillfredsställande kan förklaras enkom som en gravito-termal effekt.

    Ordet ”påvisad” infogades för att påpeka att det är upp till den som vill hävda att växthuseffekter signifikant bidrar till den 32-gradiga temperaturskillnaden att presentera belägg för att så är fallet. Att växthuseffekter skulle kunna bidra till temperaturskillnaden förefaller det mig rimligt att tro. Men jag har varken sett dig eller någon annan presentera belägg som påvisar att sådana bidrag är av signifikant storlek. Begrunda hur stort du själv anser bidraget vara, redovisa gärna grunderna för din bedömning i den mån du befinner bidraget vara av signifikant storlek, samt din därav föranledda slutsats vad beträffar IPCC:s påstående att det existerar en 32-gradig naturlig växthuseffekt.

    ”Jag kan heller inte föreställa mig att du är okunnig om vad som skulle hända om det inte funnits någon koldioxid i atmosfären enligt mycket väletablerad och 100% trovärdig forskning.”

    Vad jag är kunnig om och i mina inlägg påmint om är att man inom statistisk mekanik kommit fram till att det på grund av den gravito-termala effekten även i växthusgasfria atmosfärer kommer att uppstå en atmosfärisk temperaturgradient snarlik den som jorden reellt uppvisar, dvs att gradienten basalt återspeglar en generell gravitationseffekt utan relation till radiativa växthuseffekter.

    Vad du tycks mig syfta på är föreställningen att en atmosfär utan växthusgaser inte kan emittera termisk strålning. Den föreställningen ifrågasätts i kommentaren #10. Den förefaller även mig tvivelaktig i ljuset av Kirchhoffs strålningslag som stipulerar att all materia avger termisk strålning. Samt i ljuset av vad som empiriskt observerats: Solens fotosfär betraktas av astronomer som den lägsta delen av solatmosfären. Den innehåller inga växthusgaser. Men den avger som bekant en hiskelig massa termisk strålning, inklusive sådan inom det infraröda spektralområdet.

    Ett av skälen till att jag inte villl diskutera vad som gäller för en fiktiv Jord utan växthusgaser är att jag saknar tillräckligt djupa kunskaper om strålningsfysik för att ha något nytt av intresse att tillföra debatten. Men huvudskälet är att en sådan fiktiv Jords egenskaper snarast är av intresse för det allmänna problemet rörande växthuseffekters existens och styrka, ett problem som jag medvetet avstått från att beröra i mitt inlägg.

    Inlägget handlar om GHE-opponenters (på basis av exisensen av den gravito-termala effekten) bestridande av IPCC:s påstående att den 32-gradiga atmosfärseffekten representerar en växthuseffekt, alldenstund de finner att atmosfärseffekten kvantitativt tillfredsställande kan förklaras som en gravitationseffekt. I vilken mån även radiativt betingade växthuseffekter påverkar troposfärens och jordytans temperaturer är ett helt annat problem, vars existens jag påminner om genom att ge hänvisning till Manabes modell. Men där den för mitt nuvarande inläggs tema intressanta observationen är att även Manabes modell (för 300 ppm CO2) framställer den ca 32-gradiga temperaturskillnaden mellan utstrålningsskiktets -18 °C och jordytan som en gravitationseffekt.

    ”du bör ändra påståendet om det du bevisar till att vara villkorat av att höjden för utstrålningsskiktet är oförändrad eller liknande.”

    Självfallet bygger GHE-opponenternas argumentering på villkoret att utstrålningsskiktets höjd inte förändras (eller beaktansvärt påverkas av växthusgaser); de anser sig ju kunna beräkna denna höjd utan att ta hänsyn till växthuseffekter, och det är GHE-opponenternas synsätt jag redogjort för.

    GHE-proponenter som Wanabe anser en växthuseffekt uppkomma genom radiativa processer i översta troposfären vilka bl a får till följd att utstrålningsskiktets höjd ökar. Det är jag medveten om, men hänför denna hypotes till ett problemområde (växthuseffekters uppkomst och styrka) som jag avstått från att ta upp i mitt inlägg.

  102. Sten Kaijser

    Jag har och har läst, även om detvar ett tag sedan, Hulburts uppsats från 1931. Han argumenterar för att det är ”tropopausen” som bestämmer jordens klimat, något som ger koldioxiden en viktig roll.

    Ett ”annat synsätt” går ut på att det är värmetransporten genom avdunstning och kondensation som ger ”en lagom värme” på 5 kms höjd.

    Såvitt jag förstår är växthusgaserna oväsentliga för att förklara temperaturskillnaden mellan temperaturen vid ytan och på 5 kms höjd. Däremot är de viktiga för temperaturen på 5 kms höjd.

    Då är frågan om det räcker med vattnets värmetransport för att förklara temperaturen på hög höjd, eller om det också behövs ”långlivade växthusgaser”, som exempelvis koldioxid för att förklara den temperaturen.

  103. Thomas P

    Gösta, Kirchhoffs strålningslag säger att emission och absorption är identiska, vilket innebär att en gas som inte absorberar heller inte emitterar. Återigen en oundviklig konsekvens av termodynamikens andra huvudsats.

    Sten, så du tror inte vi skulle ha någon växthuseffekt om jorden var torr och vi inte hade någon avdunstning?

    Nog får man så många ”synsätt” på den här bloggen. Kanske borde ni börja med att enas om dessa fundamenta om hur växthuseffekten fungerar innan ni går vidare till sådant som faktiskt är svårt med klimatvetenskap.

  104. #101 Gösta Pettersson

    ”Vad du tycks mig syfta på är föreställningen att en atmosfär utan växthusgaser inte kan emittera termisk strålning.”
    Ja, exempelvis.

    ”Den föreställningen ifrågasätts i kommentaren #10.”
    ?? Syftar du möjligen på detta: ”(även icke-växthusgaser strålar, dock inte i IR-området)” I så fall är det ett missförstånd. All materia kan stråla, förutsatt att den är varm nog. Vid aktuella temperaturer strålar endast växthusgaser.

    ”Den förefaller även mig tvivelaktig i ljuset av Kirchhoffs strålningslag som stipulerar att all materia avger termisk strålning.” Nej det stämmer inte. Kirchhoffs strålningslag säger ”en ytas spektrala emissivitet är lika med dess spektrala absorptionsfaktor vid varje temperatur och våglängd.” En ideal svartkropp har e = 1 medan verkliga material har 0 < e < 1 För t.ex. kväve, syre, och argon är e=0 vid aktuella temperaturer.

    "Samt i ljuset av vad som empiriskt observerats: Solens fotosfär betraktas av astronomer som den lägsta delen av solatmosfären. Den innehåller inga växthusgaser. Men den avger som bekant en hiskelig massa termisk strålning, inklusive sådan inom det infraröda spektralområdet." Javisst. Solatmosfären 6000K, ett plasma. Den är en svartkroppsstrålare. Det är inte alls relevant för jordatmosfären.

    Definitionen på växthusgas är gas som kan absorbera IR strålning. En atmosfär som inte innehåller sådana gaser kan omöjligen stråla. Det följer av definitionen och Kirchhoffs strålningslag.

    "alldenstund de finner att atmosfärseffekten kvantitativt tillfredsställande kan förklaras som en gravitationseffekt." Det kan dom inte alls. Det är ytterst otillfredsställande att man postulerar en höjd som råkar ge rätt temperatur. Cirkelbevis skulle jag säga.

    Du skriver: "Det är nämligen gradientens lapse rate som bestämmer vilken temperaturskillnad det kommer att föreligga mellan jordytan och utstrålningsskiktet på den höjd det senare hamnar vid balans mellan instrålning och utstrålning av energi till Jorden. När IPCC och GHE-proponenter hävdar att den 32-gradiga atmosfärseffekten utgör en växthuseffekt, så är det ekvivalent med att hävda att den troposfäriska temperaturgradienten helt och hållet återspeglar en växthuseffekt." NEJ!!! påståendena är INTE ekvivalenta. Temperaturgradienten beror av den gravito-termala effekten som är oberoende av växthusgaser (i den mån inte värmekapaciteten påverkas) MEN utstrålningsskiktets höjd beror direkt av atmosfärens strålningsegenskaper.

    Utstrålningsskiktest temperatur är experimentellt känd, den ges av strålningsbalansen till 255K. Med kännedom om laps rate räknar man nedåt tills man når en höjd där temperaturen stämmer med önskat värde, 288K. Simsalabim. Nu vet vi skiktets höjd och tror att det inte skulle förändras om man t.ex. skulle släppa ut enorma mängder metan i atmosfären. Det är bara en tro – och den är gravt felaktig.

    "Självfallet bygger GHE-opponenternas argumentering på villkoret att utstrålningsskiktets höjd inte förändras (eller beaktansvärt påverkas av växthusgaser); de anser sig ju kunna beräkna denna höjd utan att ta hänsyn till växthuseffekter, och det är GHE-opponenternas synsätt jag redogjort för." OK. Då tycker jag att du skall skriva ut detta tydligt i din slutsats.

    Den som bara läser slutsatsen blir lurad anser jag. En slutsats som bygger på ett kontroversiellt antagande bör definitivt villkoras av den bygger på att antagandet är korrekt.

    "GHE-proponenter som Wanabe anser en växthuseffekt uppkomma genom radiativa processer i översta troposfären vilka bl a får till följd att utstrålningsskiktets höjd ökar. Det är jag medveten om, men hänför denna hypotes till ett problemområde (växthuseffekters uppkomst och styrka) som jag avstått från att ta upp i mitt inlägg." Detta är helt obegripligt. Hur kan du hävda att du inte behandlar "växthuseffekters uppkomst och styrka)" när du skriver: ”det existerar en gravitationsbetingad 32-gradig atmosfärseffekt som återspeglar luftmolekylernas massa och värmekapacitet, utan någon som helst påvisad kvantitativt signifikant relation till molekylernas spektrala egenskaper och förmåga att absorbera värmestrålning.” här skriver du att styrkan är 32 grader, dvs du anger styrkan. Då måste den som läser slutsatsen uppfatta din utsaga som att du anser att du visat att styrkan, 32 grader är oberoende av molekylernas spektrala egenskaper, dvs oberoende av utstrålningsskiktets höjd.

    Du skriver: "de anser sig ju kunna beräkna denna höjd utan att ta hänsyn till växthuseffekter," utan att nämna hur det går till. Dom kan inte beräkna denna höjd. Dom pillar med siffror som råkar ge ett lagom resultat. Där finns inte minsta spår av vetenskaplighet. Har du annan uppfattning i denna fråga: "hur beräkna utstrålningsskiktets höjd" skulle jag gärna vilja veta detaljerna.

  105. Gösta Pettersson

    #103
    ”Kirchhoffs strålningslag säger att …”

    Kirchhoff har gett namn till flera strålningslagar. Den jag syftade på brukar kallas hans första strålningslag. Men efter koll på nätet ser jag att den för gasformig materias del är villkorad med att gasen ska vara ”dense”. Jag är definitivt ingen expert på strålningsfysik, så jag vet inte var gränsen mellan ”dense” och ”icke-dense” går. Jag misstänker att gränsen är oskarp, så jag är ännu inte övertygad om att en växthusgasfri atmosfär omöjligen kan avge strålning.

    ”Återigen en oundviklig konsekvens av termodynamikens andra huvudsats.”

    Jag avstod från att beakta dina kommentarer föranledda det sena 1800-talets kontrovers mellan Maxwell och Loschmidt rörande den vertikala temperaturfördelningen i en innesluten luftmassa. Loschmidt hävdade att det uppstår en temperaturgradient med kallare luft i toppen. Maxwell hävdade att luftmassan skulle bli isotermisk, för eljest skulle man i strid med 2LoT få ett perpetuum mobile i form av en värmetransport från den varmare bottenluften till den kalla toppluften genom ledning i inneslutningens väggar.

    Jag tror inte Loschmidt accepterade att hans förslag stred mot 2LoT. Snarare underkände han Maxwells argument, gissningvis tankemässigt med motiveringen att gravitationen även bör påverka ledningsprocessen i inneslutningens väggar. Så att gravitationen åstadkommer omedelbart motorstopp i den värmemotor man försöker skapa genom att utnyttja de gravitationellt uppkomna temperaturskillnderna. Förvisso en intressant ”akademisk fråga”, men en fråga som jag ville undvika att länka in debatten på alldenstund den saknar relevans för det som mitt inlägg handlar om.

    Därför att rörande den icke-inneslutna atmosfären var Maxwell och Loschmidt helt överens om att det bör uppstå en vertikal temperaturgradient på grund av den gravito-termala effekten.

    ”Kanske borde ni börja med att enas om dessa fundamenta om hur växthuseffekten fungerar”

    Ack om det vore så väl att klimatologiskt vetenskap försett oss med några allmänt accepterade fundamenta om hur växthuseffekten fungerar. I stället har jag i litteraturen och klimatdebatten träffat på ett tjugotal olika hypoteser om hur det kan uppkomma växthuseffekter. Själv har jag valt att betrakta Manabes modell som ”allmänt accepterad”, men jag betvivlar att alla är eniga om att det är ett bra val.

    Och varför ska man enas? Konsensus hör politiken till. Vetenskapliga framsteg görs alltid till följd att det rått oenighet om kunskapsläget.

  106. Sten Kaijser

    Hej Thomas P,

    det var länge sedan vi utbytte tankar på den här bloggen.

    Du har visserligen alltid varit en mästare på att tolka mina eller andras kommentarer på ditt eget sätt, men jag trodde inte att vad jag skrev skulle kunna tolkas som att jag hade synpunkter på en torr jord.
    Det jag tror att de flesta är överens om är att i dagens (och gårdagens) atmosfär är det vattenångan som är den ”viktigaste växthusgasen” i den meningen att det är den som påverkar utstrålningen mest.

    Frågan är om vattenångan, utan hjälp av koldioxid skulle kunna hålla temperaturen ”beboelig”.

    Att livet ger koldioxid är en annan sak.

    Till sist Thomas, jag tycker att du tillför något till denna blogg, men du behöver väl inte alltid feltolka vad andra skriver.

  107. Sten Kaijser

    Hej Thomas P,

    jag skriver ingenting om en torr jord. Det jag noterar är att temperaturen på hög höjd främst beror på vattnets transport av värme, och den fråga jag ställer är om den förmågan skulle kvarstå även utan hjälp av koldioxid.

  108. Thomas P

    Gösta ” Jag är definitivt ingen expert på strålningsfysik, så jag vet inte var gränsen mellan ”dense” och ”icke-dense” går.”

    Kan du se igenom den strålar den heller inte i motsvarande våglängd, enligt den beskrivning av Kirschoffs lag som används i nutid.

    Jodå, Loschmidt ansåg andra huvudsatsen inte gällde, vilket inte var helt orimligt att tro då. Du har ingen temperaturgradient i väggarna. Du har inte ens samma temperaturgradient i olika gaser! Vill du offra andra huvudsatsen säg då det rakt ut.

    Den vertikala temperaturgradienten i atmosfären uppstår pga att solen värmer markytan och pga att atmosfären är ogenomskinlig för IR. Den finns inte i isolerade system.

    Konsensus är en nödvändighet för att alls kunna bedriva vetenskap. Om vi alltid måste börja från grunden skulle vi aldrig komma någon vart. Varje artikel i geografi skulle få börja med att bevisa att jorden är rund osv.

    Vill du gå emot det som är etablerad vetenskap är det upp till dig att visa att denna är fel, och det misslyckas du grovt med eftersom du gör klart att du inte förstår den. Ett resultat av att du och de andra här ägnar er tid att läsa konstiga bloggar där de mest konstiga hypoteser lanseras utan något kritiskt tänkande.

  109. Torbjörn

    #89 GoranA #93 Leif
    ”För återgå till grundfrågan som gällde om en planet utan atmosfär (A-)och en planet med atmosfär (A+) men utan växthusgaser har samma medeltemperatur.”

    Det är skillnad i instrålning mellan en planet utan atmosfär och en planet med atmosfär, pga atmosfärisk refraktion.
    Atmosfären agerar som en menisk lins som bryter solljuset mot mitten, vilket höjer intensiteten från 1370 W/m2 (egentligen 1360,8 enligt Kopp et al) till ett högre värde, gissningsvis ca 1374 W/m2 (1365).
    Det bortses från när man räknar på jordens energibalans.

    Övriga skribenter
    Växthuseffekten innefattar även moln och därmed albedot och inte bara IR strålning som en del av er tycks tro.

    När vattenångan och därmed molnen förändras, spelat det ingen roll om CO2 kan absorbera och emittera fotoner eller vilken höjd det sker på.

    Energin hos absorberade fotoner är så många gånger lägre att det drunknar i det brus/osäkerhet som beräkningar av albedot och därmed instrålningen ger.

    Dessutom förändras solinstålningen med flera W/m2 över en solcykel (just nu ca 1362 W/m2 utan hänsyn till refraktionen)

    Jorden är dessutom inte ett perfekt klot, vilket Trenberth gjort nya beräkningar på.

    När det gäller beräkningen på molekyler i ett rör ska det nog vara högre än 100m för att man ska kunna mäta skillnader i temperatur/rörelseenergi.

  110. Gösta Pettersson

    #104 Leif Åsbrink

    Beträffande Kirchhoffs strålningslagar och växthusgasfria atmosfärer se min kommentar #105

    Beträffande utstrålningsskiktets höjd säger du att GHE-opponenterna ”postulerar en höjd som råkar ge rätt svar”. Och att ”Dom pillar med siffror som råkar ge ett lagom resultat. Där finns inte minsta spår av vetenskaplighet.”

    Som jag ser det har t ex GHE-opponenten i The Hockey Schtick-artikeln inte postulerat höjden, utan klart angivit hur han beräknat den. Han må kunna ha fel i sin beräkning eller antagadena bakom den, men han har presenterat ett vetenskapligt beaktansvärt argument.

    Medan du som motargument hävdar att ”utstrålningsskiktets höjd beror direkt av atmosfärens strålningsegenskaper” utan att presentera något som helst belägg för att så är fallet. Hur ser det direkta beroendet ut, t ex i funktionellt avseende? Vilken atmosfärisk strålningsegenskap är det du syftar på? Vem är det som bygger sin argumentering på postulat à la dom har fel och jag har rätt?

    ”Utstrålningsskiktest temperatur är experimentellt känd, den ges av strålningsbalansen till 255K. Med kännedom om laps rate räknar man nedåt tills man når en höjd där temperaturen stämmer med önskat värde, 288K. Simsalabim. Nu vet vi skiktets höjd.”

    Så långt har du förstått GHE-opponenternas argument. Med reservationen att 288K inte avser något av dom önskat värde utan ett empiriskt kalkylerat värde. Men sen fortsätter du

    ”Nu vet vi skiktets höjd och tror att det inte skulle förändras om man t.ex. skulle släppa ut enorma mängder metan i atmosfären. Det är bara en tro – och den är gravt felaktig.”

    GHE-opponenternas av mig redovisade argumentering uttrycker inte någon sådan tro. Det är du själv som med insikt om implikationerna av GHE-opponenternas slutsats rörande den naturliga växthuseffektens signifikans uttrycker din egen trosvissa övertygelse om att det finns en förstärkt växthuseffekt av signifikant storlek. Därför måste dom ha fel och du ha rätt.

    ”Detta är helt obegripligt. Hur kan du hävda att du inte behandlar ”växthuseffekters uppkomst och styrka …”

    Därför att jag inte gör det. Inlägget handlar inte om växthuseffekten utan om den gravito-termala effekten och konsekvenserna av den senare effektens existens. Främst med avseende på IPCC:s påstående att Jorden uppvisar en 32-gradig naturlig växthuseffekt.

    Jag tror det är dags för oss två att enas om att vi inte är överens i alla avseenden. Jag har sagt vad jag ville ha sagt. Du har fått tillfälle att med anledning därav säga vad du vill ha sagt.

  111. Gösta Pettersson

    #108

    Vi har nog vitt skild syn på vad som ska betraktas som etablerad vetenskap vad växthuseffekten beträffar. Så det förefaller mig vara dags att enas om att vi i det avseendet inte är överens om särskilt mycket.

  112. Thomas P

    Gösta, du har nog rätt. De som en gång gått med bland ”drakdödarna” återvänder inte till verkligheten. du får bortse från energikonservering, termodynamikens grundläggande lagar etc. Det är ju i grunden de här som gärna hänvisar till dig som auktoritet som det är mest pinsamt för.

  113. Munin

    Gravitation, denna urkraft, håller planeterna på plats i vårt solsystem och skapar en stor stabilitet i hur även jorden exponeras för solens utgående energiflöden.

    Jordens egen gravitation verkar på all massa i sin egen lilla sfär och därmed återverkar den på hur massan ordnar sig ända ut till atmosfärens yttre gräns.

    Det är svårt att tänka sig annat än att gravitationen påverkar hur atmosfären skiktar sig och måste vara med för att kunna förklara varför ämnen (fasta, flytande, gaser) finns där de gör.

    Gravitationen borde vara en grundbult i klimatmodellerna och inte minst för konvektionen och vattnets rörelser. Det är svårt att föreställa sig hur det någonsin skulle kunna modelleras utan att ta med gravitationen.

    En enkel fråga: Var befinner sig alla de klimatmodeller, som nu torgförs? Hur ingår gravitation i dessa? Nederbörd i form av lätta och tunga regn ges gravitationen någon roll? Etc….

  114. #100 Adepten

    Du skriver: ”Du leder i bevis att Görans teori är falsifierad genom att utgår från att Jorden är en platt skiva, det imponerar inte på mig” Din förmåga att läsa innantill imponerar inte på mig….

    Göran säger att han utgår från Lamberts cosinuslag. Den säger att den strålningsintensitet eller ljusintensitet som observeras från en ideal diffust reflekterande yta eller ideal diffus radiator är direkt proportionell mot cosinus av vinkeln θ mellan riktningen för det utgående ljuset och ytnormalen.

    Det gör han inte alls. (Även om formeln även gäller för absorption av infallande ljus mot svartkropp.)

    Det är mycket enklare. En av fysikens grundpelare, termodynamikens första huvudsats (energiprincipen) som säger att energi varken kan skapas eller förstöras ger formeln.

    En plan yta vinkelrätt mot infallande strålning ger en skugga som är direkt proportionell mot ytan A.

    I ett ljusflöde med intensitet 1370 W/m2 mottager ytan A kvadratmeter 1370A W = 1370A J/s. I ett plan bakom ytan har ljusflödet minskat med 1370 J/s enligt första huvudsatsen. (Jag skriver Joule/sekund i stället för watt för att tydliggöra att det är energin – varje sekund – som är oförstörbar.)

    Vrider vi ytan så att vinkeln mellan ytnormalen och infallande strålning blir v kommer skuggans yta att minska till A*cos(v) och därvid kommer den energi som fattas i planet bakom att bli 1370Acos(v) J/s

    Vi behöver inte bekymra oss om lambertiska ytor sådana som Lamberts cosinuslag handlar om.

    I specialfallet cirkulär skiva är A=pi*r^2

    Om vi i stället tittar på skuggan av ett klot så är skuggans area A=pi*r^2 Klotets area är 4pi*r^2

    Den energi som mottagits av klotet måste vara 1370*pi*r^2 J/s Det är den energi som fattas i planet bakom klotet. Denna energi är naturligtvis ojämnt fördelad över klotet. Det vi vill veta är det genomsnittliga energiflödet från varje kvadratmeter av klotets yta när det totala energiflödet fördelas lika över hela ytan. Det blir 1370/4*pi*r^2 Klotets yta är 4 gånger större än skuggans yta.

    Görans modell M1 (=IPCC) är således korrekt.

    Dubbelintegraler är besvärligt. Jag vet inte hur man löser det här problemet analytiskt. Möjligen kunde jag sån här matematik för 50 år sedan men jag har glömt ifall de kurser i funktioner av flera variabler som jag läst räckte för sådana här problem.

    I själva verket är det egentligen ett vanligt fel Göran begår. Han har ett fysiskt problem men misslyckas med att formulera det matematiskt.

    För att sätta upp en korrekt formel betraktar vi en yta på den solbelysta halvan med koordinaterna lat och long. Dela in ytan i rutor med storlek 180/n grader i både latitud och longitud. Betrakta situationen när solen står i zenith vid koordinaterna lat=0 och lon=0 dvs på ekvatorn och Greenwichmeridianen.

    När vi från punkten (lat,lon) betraktar solen finner vi att cosinus för vinkeln mellan zenithriktningen och solen är cos(lat)*cos(lon) formler här: http://www.fnf.nu/_filer/bagerfeldt/46%20Riktningar%20mot%20Himlavalvet%20-%20Passager%20och%20uppg%C3%A5nga

    En rutas storlek i latitudled pi*r/n och i longitudled cos(lat)*pi*r/n När vi summerar alla ytor med hänsyn till solens höjd över horisonten får vi problemet att integrera cos(lat)*cos(lon)*(pi*r/n)*(cos(lat)*pi*r/n) för lat och long i intervallet -90 till +90 grader.

    Av det sätt du kritiserat mig på förmodar jag att du vet hur man räknar ut sådana här integraler så jag föreslår att du förvissar dig om att resultatet blir samma som i M1.

    Om du liksom jag finner det besvärligt kan du ju betrakta följande lilla programsnutt och eventuellt provköra den:

    {
    double a, pi, r, lat, lon, sum, p;
    int n;
    n=1000; //antal steg
    pi=3.14159265359;
    lat=-(pi+pi/n)/2;
    sum=0;
    r=3670000; //jordradie i meter
    for(i=0; i<n; i++)
    {
    lat+=pi/n;
    lon=-(pi+pi/n)/2;
    for(j=0; j<n; j++)
    {
    lon+=pi/n;
    sum+=cos(lon)*cos(lat)*cos(lat);
    }
    }
    p=1370;
    a=4*pi*r*r;
    // multiplicera med konstanterna
    printf("\nklot %f",pow((pi*r/n),2)*sum*p/a);
    printf("\nplatta %f\n\n",pi*pow(r,2)*p/a);
    }

    Så här blir resultatet på min maskin:

    klot 342.500141
    platta 342.500000

    Jag hoppas du, Göran Åkesson läser denna kommentar i sin helhet och att du därefter finner det lämpligt att se till att det inlägg på klimatsans som du nu borde förstå är felaktigt blir borttaget.

  115. Henrik Svensson, vd för det oljeeldade reservkraftverket i Karlshamn skriver larmartikel i Dagens Industri.
    https://www.di.se/nyheter/larmet-valdigt-allvarligt-lage-for-elforsorjningen/

    Det är totalt oansvarigt enligt min mening att låta CO2-skräcken förhindra att man i tid ser till att Sverige kan producera el även ifall det blir en mycket kall vindstilla vecka kommande vinter. Vindstilla i våra grannländer också. Eventuellt stopp på gas från Ryssland. SVK har roterande bortkoppling som sista utväg för att se till att elnätet inte kraschar. Skulle dom misslyckas och vi får en blackout kan det ta en vecka att få strömmen tillbaka. Vad som då skulle hända tror inte jag någon planerat för. Moderna hus med lätta väggar och god isolering kallnar fort ifall värmekällan stängs av. Är värmedistributionen till olika rum vattenbaserad måste allt vatten tappas ur elementen rätt snart. Även dricksvatten, diverse vattenlås och annat måste tömmas. Detta fattar inte ansvariga omedelbart eftersom strömavbrott inte är någon extrem händelse normalt sett. Jag tror moderna fastighetsförvaltare inte längre har en portvakt/fastighetsskötare i varje fastighet. Personal från något servicebolag får åka runt och fixa. Hur gör dom när det inte går att tanka? Och telefonnätet havererat? Det skulle kunna gå väldigt illa. Att se till att (den inte särskilt stora) risken för blackout minimeras ser jag som mycket viktigt eftersom konsekvenserna skulle kunna bli så katastrofala.

    I dag premierar energibolagen värmeproduktion och el kommer i andra hand. Detta kan ändras och jag tror man kommer att behöva tillskott av fossil energi i kraftvärmeverken. Då behöver diverse lagar ändras. Så fort detta börjar diskuteras kommer de mest militanta alarmisterna att försöka stoppa ”nya fossila utsläpp.” Det kan bli en infekterad debatt – och i den kommer personer som läst ”att det existerar en gravitationsbetingad 32-gradig atmosfärseffekt som återspeglar luftmolekylernas massa och värmekapacitet, utan någon som helst påvisad kvantitativt signifikant relation till molekylernas spektrala egenskaper och förmåga att absorbera värmestrålning.” att sprida denna falska information: ”det är bevisat att CO2 inte har någon inverkan på jordens temperatur.” Även dom som läst ”Solinstrålningen räcker till för att värma upp jordklotet – växthusgaseffekten kan inte existera” kommer att sprida samma falska information.

    Sådant sprids ju då och då på debatt och insändarsidor och det eldar på alarmisterna som får skäl att misstro alla som är emot deras agenda. Jag hoppas innerligt att det skall bli offentlig debatt om eldandet av sopor och fossila bränslen i kommande valrörelse. Alarmister, ”klimathotsreligiösa” som det ofta heter här på KU är människor som på goda grunder blivit uppskrämda av alla alarmartiklar. Hur skall dom kunna veta att det mesta är bluff och båg? Det behövs en trovärdig motberättelse. Den saboteras av vad jag skulle vilja kalla ”klimathotsförnekelsereligiösa” som läser på ”skeptikersidor” att CO2 saknar betydelse – eller ännu värre, växthusgaser saknar betydelse för klimatet. Det ”religionskrig” som riskerar att blomma upp kommer att engagera de uppskrämda alarmisterna, ge Extinction Rebellion och Fridays for Future med flera luft under vingarna. Deras positioner kommer att stärkas av de uppenbart falska påståenden som frekvent förekommer i skeptikerkretsar. ”Klimathotsförnekelsereligiösa” anamar kritiklöst varje påstående som säger att CO2 inte påverkar klimatet. Jag har försökt påvisa två flagranta fall i kommentarerna till det här inlägget.

    Visst kan man ifrågasätta IPCC:s ”bevis” för att det är växthusgaserna som förorsakat den observerade temperaturstegringen. Något strikt bevis föreligger inte. IPCC anför att modeller som inte tar hänsyn till den antropogena CO2-ökningen inte kan fås att stämma med observationer. Det är inget bevis. Jag är övertygad om att det går att konstruera modeller där hela uppvärmningen har någon annan orsak. Typ: ”Solens väg genom rymden har ändrat den kosmiska omgivningen” eller vad som helst som varken kan bevisas eller motbevisas.

    Däremot finns spektra av strålningen från jorden. Vad som skulle hända om CO2 helt försvann är enligt min mening påvisat trots att experimentet inte kan utföras praktiskt. Den samlade kunskapen om strålningsfysik ger oss en säker bild av vad som skulle ske om CO2 momentant försvann ur atmosfären.

    Betänk hur landet ligger. För ett par dagar sedan hade DN en skräckartikel. Alldeles vanlig i dagens galna värld: ”I Indien faller uttorkade fåglar från himlen. Visst känns det normalt? Temperaturen har under årets historiska värmebölja närmat sig 50 grader. När asfalten smälter, soptippar självantänder och skördarna krymper kan du tänka på det som en filmtrailer. I mitten av maj släppte brittiska Met office en studie som visar hur klimatförändringarna driver på intensiteten i värmeböljorna, vilket gör rekordartade temperaturer ’hundra gånger’ mer sannolika.” Detta är vad svenska folket matas med dagligen: https://www.dn.se/kultur/kristina-lindquist-det-vidriga-kriget-blir-en-perfekt-forevandning-for-att-ignorera-klimatkrisen/

    Att det ur utsläppssynpunkt vore bättre att elda kol i svenska kraftvärmeverk än att importera el från tysk kolkondens behöver väl inte diskuteras. Lägre överföringsförluster och tillvaratagande av värmeenergi. Men Sverige styrs av ett klimatmål med syfte att minimera utsläpp inom Sveriges gränser. Med tanke på den tid det skulle ta för kommuner och andra som har kraftvärmeverk att ställa om och eventuellt ändra personalbemanningen för att prioritera el behöver lagändringar ske mycket snabbt. Det finns kraftvärmeverk som inte gör någon el alls. Medelpriset är för lågt och dom får inget för att dom kan producera dom relativt korta tiderna när vindkraften inte räcker.

    Nu har vi den olyckliga situationen att det byggs mer vindkraft än vi har balans och reglerkraft till i Sverige. Vad vi kan göra är att först och främst se till att befintliga resurser används optimalt.

    Jag anser att prioritet ett är att gära kraftvärme mer lönsamt. Bort med avfallsförblänningsskatten. Elcertifikat för regler och balanskraft. Uppmuntra (ekonomiskt) att man bygger värmelager vid kraftvärmeverken. Det är väldigt mycke enklare och billigare att lagra värme än att lagra el. Kraftvärmeverk kan installera stor värmepump och i värmelager spara energin från den överskottsel som för mycket vindkraft tillför systemet.

    Om Sverige satsar på att optimera fjärrvärmeverken och gör det attraktivt att även använda fossila bränslen skulle vi lösa en betydande del av de problem som vind och sol förorsakar. Problemen försvinner inte för att vi bygger kärnkraft. Det skulle bli vansinnigt oekonomiskt med kärnkraft som bara behövs när det inte blåser. De enda som skulle ha råd att bygga under sådana omständigheter är du och jag (skattebetalarna.)

    Jag hoppas att jag med denna kommentar har tydliggjort varför jag här på KU så ofta kritiserar ”skeptiska” skrifter som jag anser är vetenskapligt felaktiga.

    För övrigt rekommenderar jag varmt att alla läser Lennart Bengtssons bok:
    VAD HÄNDER MED KLIMATET.

  116. #14 Ingemar Nordin

    1) Alltså är lapsraten en gravitationell effekt. JA!

    På den andra sidan finns strålningsfysik och kvantmekanik: Enligt dessa så finns det en påvisbar uppvärmningseffekt av växthusgaser som har en förmåga att absorbera och emittera strålning. JA

    Den förmågan är som störst vid små halter av t.ex. CO2, mellan 0 och 300 ppm. Därefter avtar effekten. Effekten följer en logaritmisk kurva. JA

    Alltså MÅSTE en DEL av lapsratens utseende i jordens atmosfär vara en konsekvens av existensen av växthusgaser.?????

    Vadå alltså? Lapsraten är oberoende av CO2-halten (i fallet att luftfuktigheten inte påverkas)

    Mer CO2 innebär att strålningen från CO2 kommer från högre höjd där det är kallare. Den bär då mindre energi med sig. Det finns mer CO2 på hög höjd som strålar nedåt och gör att luften på lägre höjd kyls långsammare.

    ”Jag får inte de båda delarna av välkänd fysik att gå ihop. Någon har räknat fel någonstans.” Hur? Jag förstår inte vad du finner konstigt.

  117. Gösta Pettersson

    #104 Leif Åsbrink

    Hej Leif. Fortsätt att kolla kommentarerna till mitt inlägg. I morgon (dvs senare i dag) ska jag förklara vad jag så här på kvällskvisten inser vara den egentliga och eliminerbara orsaken till skillnaden i våra synsätt.

  118. Adepten

    113 Leif Åsbrink

    👍🤩🧐 Du är ett ljus i mörkret!

  119. KM

    #116 Leif Åsbrink

    ”Lapsraten är oberoende av CO2-halten (i fallet att luftfuktigheten inte påverkas)”

    Men IPCC m.fl. påstår ju att vattenånga (d.v.s. luftfuktighet) stiger som respons på stigande CO2-halt och att detta ger upphov till en ”förstärkningseffekt” som krävs för att förklara den påstått höga klimatkänsligheten för CO2.

    Men detta skulle ju göra så att lapsraten minskar, vilket i sin tur skulle innebära att temperaturen vid jordytan sjunker.

    En negativ återkoppling alltså som IPCC verkar helt ha bortsett ifrån?!

  120. Sten Kaijser

    Hej KM #119

    om temperaturen på en viss höjd är bestämd och lapse rate sjunker så blir det varmare ”där nere”.

  121. KM

    #120 Sten Kaijser

    Tror inte det!

    ”Temperaturen på en viss höjd”, säg 5 km, är väl 255 K d.v.s. -18 °C.

    Temperaturen vid ytan är varmare p.g.a. lapse rate, ca 32 °C enligt GPs förklaring (+6,5 °C/km x 5 km).

    Om lapse raten sjunker till t.ex. 6 °C/km så skulle väl ytan jämförelsevis bli ca 2,5 °C kallare.

    Däremot skulle det bli motsvarande nycket varmare i toppen av troposfären.

    Eller?

  122. Gösta Pettersson

    #104 Leif Åsbrink

    Jag gav upp i min kommentar #110 eftersom jag inte begrep varför du bestred vad som för mig är rena självklarheter.

    T. ex. ”NEJ!!! påståendena är INTE ekvivalenta. Temperaturgradienten beror av den gravito-termala effekten som är oberoende av växthusgaser (i den mån inte värmekapaciteten påverkas) MEN utstrålningsskiktets höjd beror direkt av atmosfärens strålningsegenskaper..”

    Där din anmärkning om utstrålningsskiktets höjd ledde in mig på fel tankebanor och irrelevanta resonemang om vad som bestämmer utstrålningskiktets höjd. Nu har jag haft ytterligare ett dygn att grunna på hur du har tänkt och omsider insett vad som som tycks mig vara en helt avgörande skillnad mellan ditt och mitt synsätt.

    Jag, de GHE-opponenter vars argumentering jag redogjort för, meteorologer i gemen, samt kreti och nästan alla pleti ansluter oss till Maxwells förmodande att troposfärens temperaturer på grund av den gravito-termala effekten basalt strävar mot och i det närmaste uppnår ”en konvektiv termisk jämvikt” karakteriserad av Loschmidts ekvationer för den uppstådda temperaturgradienten. Av den enkla anledningen att sagda förmodande ger en tillfredssställande förklaring till varför den observerade temperaturgradienten ser ut som den gör.

    Enligt denna Maxwellska syn spelar det ingen roll i vilken mån radiativa effekter bidrar till värmetransporten från jordytan till utstrålningsskikten eller på annat sätt tenderar att påverka de troposfäriska temperaturerna. Slutresultatet blir att man hamnar i den ”konvektiva jämvikten”.

    Må vara att växthusgaser och radiativa processer ger värmebidrag som t ex skulle kunna förhöja utstrålningsskiktets höjd vid radiativ jämvikt. Men grundtanken bakom den ”konvektiva jämvikten” är att någon radiativ jämvikt aldrig hinner upppnås eftersom konvektionen ger snabbare energitransport och hela tiden omfördelar temperaturerna mot den Loschmiska temperaturgradienten som är av rent gravitationellt ursprung.

    Med andra ord, i meteorologernas standardbeskrivning av den troposfäriska temperaturgradienten (linjär, når upp till troposfären) ingår som en grundförutsättning att det råder ”konvektiv jämvikt” så att radiativa bidragstendenser är utsuddade av konvektionen. Det är med utgångspunkt från denna standardbeskrivning som GHE-opponenterna kommer fram till att den 32-gradiga atmosfärseffekten omöjligen kan vara en växthuseffekt utan tillfredställande kan förklaras som en gravitationseffekt.

    Cirkelbevis? Nej man ska väl snarare se det som att föreställningen om den ”konvektiva jämvikten” vuxit fram för att den gett en tillfredsställande god förklaring av temperaturgradientens observerade egenskaper.

    Hoppas detta förtydligande ska öka förståelsen av hur GHE-opponenterna resonerar.

  123. JonasW

    #59 Sten

    Mycket bra fråga. Om man har ett rör som går ut till rymden – hur kommer tryck, täthet och temperatur att fördela sig i detta rör?

    Svaret är att detta är ett olöst problem. Man kör fast med ”vanlig” termodynamik, och man kör fast med statistisk mekanik.

    Man kan tycka att ett så ”enkelt” problem borde vara löst, men det är det inte. Problemet är hur man ska infoga gravitationen i statistisk mekanik. Än finns ingen bra modell.

    Det finns säkert läsare som stött på räkneexempel där man räknar ut tätheten i ett rum med typ 2-3 meters takhöjd och kommer fram till att tätheten (och temperaturen) avtar exponentiellt. Det man gör då är att man byter ut uttrycket för kinetisk energi till kinetisk + potentiell energi (utan motivering).

    Idag finns ingen teoretisk modell som beskriver hur temperatur, täthet och tryck ser ut i ett långt rör från jordytan och uppåt.

  124. Thomas P

    #119 KM, det du beskriver kallas för ”lapse rate feedback”. Den är mycket riktigt negativ och den finns med i modellerna och nämns i IPCC:s rapporter. Se t ex WG1 AR5 Tabell 9.5.
    (Inte alla modeller separerar effekterna så de kan anges var för sig, därav alla ”n.a”)

    #122 GP Det vore nog bra om du beskrev hur du kommer fram till effektiva höjden på utstrålningen om du menar att växthusgaser inte har något med saken att göra. Tror du fortfarande att en atmosfär utan växthusgaser kan stråla ut IR?

  125. Lars-Eric Bjerke

    #123 JonasW,
    ”Idag finns ingen teoretisk modell som beskriver hur temperatur, täthet och tryck ser ut i ett långt rör från jordytan och uppåt.”

    I det enklaste fallet d.v.s. i det adiabatiska röret är modellen mycket enkel.
    -Temperaturen blir densamma i hela röret oberoende av gravitationens storlek enligt termodynamikens andra huvudsats.
    – Tätheten blir en funktion av gravitationens storlek. Om gravitationen är noll blir den likformig i hela röret, om den är mycket hög blir densiteten mycket hög i botten av röret.
    – Trycket fördelar sig som m*g*h.

    Johan M. modell visar också detta. Om laps rate är oberoende av gravitationen i det adiabatiska fallet, följer att den är det även i övriga fall på vår jord. Det är inte svårare än så enligt min uppfattning.

  126. Sten Kaijser

    KM #21

    Du har alldeles rätt!

  127. Adepten

    #114 Leif Åsbrink

    Tack för det matematiska beviset 🙂 Men varför använder du en iteration på n=1000?
    Varför inte 90 eller 180? Du använder dig av C++ antar jag. Min dator har inte begåvats med matteprogram annat än de som finns i excel 🙁 Så om du kunde omvandla beviset till Excel vore jag mer än tacksam 🙂

  128. Adepten

    Gösta Pettersson

    Stort tack Gösta Pettersson för ditt inlägg som skapade rekordmånga kommentarer av vitt skilda slag 🙂
    Den leder också in på den Global värmebalansen och hur den beräknas . Där man antar att instrålad energi på ca1370 W/m2 reduceras till ca 340 W/m2 vid tropikerna pga reflektion, spridning och absorption. Hur man nu kan bestämma det så noggrant?

  129. Håkan Bergman

    Adepten #127
    Python är fritt och finns för alla system, jag tog mig friheten att skriva Leifs program i python det ser ut så här:
    import math
    n = 1000
    a = 0.0
    r = 0.0
    lat = 0.0
    lon = 0.0
    i = 0
    j = 0

    pi = 3.14159265359
    lat = -(pi + pi / n) / 2
    sum = 0.0
    r = 3670000.0
    for i in range(n):
    lat += pi / n
    lon = -(pi + pi/ n) / 2
    for j in range(n):
    lon += pi / n
    sum += math.cos(lon) * math.cos(lat) * math.cos(lat)
    p = 1370
    a = 4 * pi * r * r
    print(’klot’, math.pow(( pi * r / n), 2) * sum * p / a)
    print(’platta’, pi * math.pow(r, 2) * p / a)

    Jag får följande:
    klot 342.5001408475096
    platta 342.5

  130. Håkan Bergman

    Jaha det blev ju inte så bra, WP tog bort alla indragningar, men man kan hämta filen här:
    https://sprayhattas.crabdance.com/klotplatta.py

  131. Adepten

    #130 Håkan Bergman

    Tack för tipset 😊
    Men varför använder du en iteration på n=1000? Varför inte 90 eller 180?

  132. Håkan Bergman

    Adepten #131
    För att jämföra med originalet så klart, jag får följande.
    n = 1000
    klot 342.5001408475096
    platta 342.5

    n = 90
    klot 342.5173891956791
    platta 342.5

    n = 180
    klot 342.5043471830313
    platta 342.5

  133. Adepten

    #132 Håkan Bergman

    Med risk för att verka tjatig 😉
    Kan du ge uppgifter på n=1 n=10 och n=45.

    Jag installerade Python men får bara syntaxError ! Har inte tid att kolla detta nu.
    File ””, line 7
    p = 1370
    ^
    SyntaxError: invalid syntax
    >>> a = 4 * pi * r * r
    >>> print(’klot’, math.pow(( pi * r / n), 2) * sum * p / a)
    Traceback (most recent call last):
    File ””, line 1, in
    NameError: name ’p’ is not defined. Did you mean: ’pi’?
    >>> print(’platta’, pi * math.pow(r, 2) * p / a)o

  134. Håkan Bergman

    Adepten #133
    1
    klot 1075.995483854575
    platta 342.5

    10
    klot 343.9125398542178
    platta 342.5

    45
    klot 342.5695641995787
    platta 342.5

    Som du ser så konvergerar klot och platta med fler iterationer.
    Python är verkligen petigt när det gäller layouten av koden, men det lär man sig och uppskattar efter ett tag. Det räcker med ett mellanslag mer eller mindre i början av raden så får du fel.
    Om du kör windows så ska idle ingå i installationen, testa att öppna och köra filen med idle, den kommer att visa eventuella syntaxfel.

  135. Adepten

    # 134 Håkan Bergman
    OK ! får samma resultat som dig 🙂
    Hittade felet 🙂

  136. #122 Gösta Pettersson

    Hej Gösta! Jag håller helt med om detta: ”Enligt denna Maxwellska syn spelar det ingen roll i vilken mån radiativa effekter bidrar till värmetransporten från jordytan till utstrålningsskikten eller på annat sätt tenderar att påverka de troposfäriska temperaturerna. Slutresultatet blir att man hamnar i den ’konvektiva jämvikten’ ”.

    Följande resonemang är däremot alldeles fel i mina ögon: ”Må vara att växthusgaser och radiativa processer ger värmebidrag som t ex skulle kunna förhöja utstrålningsskiktets höjd vid radiativ jämvikt. Men grundtanken bakom den ’konvektiva jämvikten’ är att någon radiativ jämvikt aldrig hinner upppnås eftersom konvektionen ger snabbare energitransport och hela tiden omfördelar temperaturerna mot den Loschmiska temperaturgradienten som är av rent gravitationellt ursprung.”

    Begreppet ”den konvektiva jämvikten” är ofullständigt. Hela atmosfären är alltid i ”radiativ-konvektiv” jämvikt på våglängder där absorptionen är så hög att en foton inte har någon chans att nå så långt att den kan nå en avvikande temperatur. Wijngaarden & Happer uttrycker saken så här: ”The change in surface temperature due to CO2 doubling is estimated taking into account radiative-convective equilibrium of the atmosphere”

    Detta må förefalla abstrakt, men jag kan ge några exempel:

    Först gällande atmosfären nära marken med normal koldioxidhalt. Transmissionen som funktion av frekvensen ges av figur 6.1 här: http://irina.eas.gatech.edu/EAS8803_Fall2009/Lec6.pdf övre kurvan. Strålning vid centrum av den starkaste absorptionstoppen 667.5 cm-1, försvagas med faktorn 25 från 1 till 0,04 efter en sträcka av 1 m. Dämpningen är logaritmisk, halvering på avstånd x får vi ur x=log(2)/log(25) vilket ger x=0,21 m. Vi får denna tabell:

    Avst Effekt
    (m)
    0.000 1
    0.210 0.5
    0,420 0.25
    0,630 0,125
    1,000 0,037
    2,000 0,0014
    3,000 0,000052
    4,000 0,0000019

    Tänk nu att vi skaffar ett infrarödteleskop med spektrometer och effektmätare. Vi ställer in frekvensen 667,5 cm-1 (20011.0 GHz) och väljer brusbandbredd 1 GHz (0,033 cm-1). Riktar vi teleskopet uppåt kommer vi att att avläsa effekten av en svartkroppsstrålare inom bandbredden1GHz ifall vår effektmätare är ideal och vårt teleskop förlustfritt. (det får alltså inte finnas någon CO2 inuti teleskopet och speglarna måste vara ideala.) Effekten vi ser kallar vi P, enhet W

    Om vi placerar en svartkropsstrålare med mycket högre temperatur, cirka 800 K omedelbart framför teleskopet blir effekten tio gånger högre, 10*P W

    Sätter vi den 10 gånger starkare svartkroppsstrålaren på ett avstånd av 4 meter kan vi inte registrera att den finns, vi ser bara luftens 14 °C. På avståndet 1 m ser vi (0,037*10*P + (1-0,037)*P) = 1,333*P W

    Det vi gör i detta tankeexperiment har en konkret motsvarighet vid mikrovågsfrekvenser där dämpningen i atomsfären och det termiska bruset från atmosfären sätter gränsen för räckvidden. På mikrovågsfrekvenser är svartkroppsstrålningen proportionell mot temperaturen. I radiosammanhang talar man om brustemperatur och den är additiv.

    Mekanismen är att atmosfären vid temperaturen T exciterar CO2-molekyler genom kollissioner. För enkelhetens skull approximerar jag CO2 med en molekyl med endast två nivåer och då ger Boltzmannfördelningen att vid 287K befinner sig ungefär 5% av alla CO2-molekyler i sitt exciterade tillstånd. Dom allra flesta de-exciteras genom kollissioner med luftmolekyler, företrädesvis kväve varvid excitationsenergin övergår i kinetisk energi (värme.) Någon gång ibland sker de-excitation genom utsändande av en foton. En sådan foton har 50% sannolikhet att absorberas inom en sträcka av 0,21 meter. Ifall en foton råkar träffa en exciterad CO2-molekyl kan den exciterade de-exciteras, stimulerad emission.

    Det elektromagnetiska fältet befinner sig i radiativ-konvektiv jämvikt inom det betraktade frekvensområdet. Ingen energi slipper ut till luft med avvikande temperatur, men hela luftmassan kan röra sig. Konvektion kan pågå, vilket ju är en förutsättning för den konvektiva termiska jämvikt som Maxwell talar om.

    På grund av tryckbreddning är hela området 650 till 690 cm-1 helt ogenomskinligt vid marknivå med transmission under 0,25 på en sträcka av 100 m.

    Avst Effekt
    (km)
    0.000 1
    0.100 0.25
    0.200 0.0625
    0.400 0.0039

    Flyttar man IR-teleskopet till höjden 100 hPa blir situationen en annan. Spektrallinjerna är mycket smalare på grund av det lägre trycket och även den lägre temperaturen. Mellan spektrallinjerna är transmissionen nära 1,0 utom runt 667 cm-1 där linjerna ligger mycket tätt. Mäter man uppåt blir effekten långt under vad som motsvarar svartkroppsstrålning från omgivande atmosfär. Den yttre rymdens temperatur, 3K skiner igenom. Då råder inte längre radiativ-konvektiv jämvikt inom det betraktade frekvensområdet. Atmosfären förlorar fotoner och därmed energi. Det är en kylande effekt. Den balanseras genom tillförsel av värme underifrån via konvektion.

    Eftersom jag inte förstår ditt resonemang undrar jag hur det påverkas när du inte kan åberopa att konvektionen ger snabbare energitransport.

    Summerar man över hela IR-spektrum sker det ett läckage av strålningsenergi från alla höjder ända från markytan och högt upp i stratosfären. Varje gång en foton råkar sändas ut på någon frekvens där absorptionen uppåt är tillräckligt svag finns en chans att nå universum. Kylningen sker alltså inte alls från någon bestämd höjd, ”det utstrålande skiktet” är bara ett medelvärde av energiflödet över frekvens och rymdvinkel.

  137. Munin

    Var landar alla de klimatmodeller (hundratals finns), som påstår sig klara av att förutsäga klimatet. Hur ska de klassificeras? Vilken jämvikt står de för?
    • Konvektiv jämvikt
    • Radiativ-konvektiv jämvikt
    • Radiativ jämvikt

    Vilka klimatmodeller har med gravitationen och vilka hävdar att den inte återverkar på hur klimatet blir? Tråden har verkligen satt fokus på gravitationen!

  138. bength

    #78 Gösta Pettersson
    ” Jag känner till hypotesen att växthusgaser förhöjer ”utstrålningsskiktets” läge, men har inte träffat på några empiriska eller modellmässiga belägg för att så är fallet.”

    Att utstrålningshöjden beror av mängden växthusgaser, dvs gaser som absorberar/emitterar i IR-spektrum (ca 2.5–50 mikrometer), kan väl knappast sägas vara en ”hypotes”, utan kan härledas från känd och allmänt accepterad strålningsfysik.

    Sannolikheten för att en foton som emitterats i ett absorptionsband av en molekyl i atmosfären ska stråla ut till rymden utan att absorberas av en annan molekyl av samma slag ökar med höjden, eftersom antal molekyler ovanför som kan absorbera minskar med höjden från vilken fotonen emitteras.

    Ökas tätheten av dessa molekyler så hittas motsvarande sannolikhet på en högre höjd. Utstrålningen till rymden kommer då att ske från högre höjder. Och utan växthusgaser i atmosfären så skulle utstrålningshöjden bli 0 m, dvs utstrålningen skulle ske från jordytan för alla våglängder, på samma sätt som redan sker för våglängderna i det s k atmosfäriska fönstret.